Book 4 -- Tax Planning

Ace your homework & exams now with Quizwiz!

In 2019, Jan inherited stock valued at $4,000 from her father. Her father originally purchased the stock for $500. She sold the stock 6 months after her father's death for $6,000. What gain did Jan recognize on the sale of this stock? A)$2,000. B)$5,500. C)$6,000. D)$4,000.

A // A person who inherits property receives a stepped-up basis equal to the value of the property at the decedent's death. Jan's basis for the sale was $4,000. Her gain on the sale was $2,000 ($6,000 − $4,000). Because inherited property is deemed to be held long-term, Jan's gain is taxed at the long-term capital gain rate.

In Year 1, Woody loaned his friend, Wesley, $3,500 to help Wesley pay his creditors. Wesley was to repay the loan by the end of Year 2. Instead of paying his creditors, Wesley laid the entire loan down on one roll of the dice at the local casino and lost. In Year 3, after filing for bankruptcy, Wesley could repay Woody only $250. What is Woody's realized loss in Year 3 as a taxpayer with a single filing status? A)$3,250 short-term capital loss in Year 3. B)$3,500 short-term capital loss in Year 3. C)$3,000 short-term capital loss in Year 3 and $250 short-term capital loss in Year 4. D)$3,000 ordinary loss in Year 3.

A // This loan is considered a nonbusiness bad debt. Nonbusiness bad debts are considered short-term capital losses in the year in which they become completely worthless. Woody will realize a short-term loss of $3,250 in Year 3 ($3,500 loan − $250 actually paid). Assuming Woody had no gains during the year, he will be able to recognize a $3,000 short-term capital loss in Year 3 and will carry over the remaining capital loss of $250 to the next year. The $3,000 short-term capital loss in Year 3 and $250 short-term capital loss in Year 4 are recognized, not realized, losses.

ABC, Inc., is an S corporation. Bill and Diane are married and the sole owners of ABC. They are 2 of the 20 employees of the firm. ABC provides each of its employees $50,000 of group term life insurance coverage. The premiums on what total amount of insurance coverage are taxable to Bill and Diane? A)$75,000. B)$100,000. C)$50,000. D)$0.

B

Dr. Argon is a dentist who operates as a sole proprietor. He bought a sterilization machine for $5,000. He used the MACRS 7-year percentages to recover the cost of the machine. Dr. Argon sold the machine in the current year for $3,000. He has claimed $1,200 of cost recovery deductions. How much will his gain or loss be in the current year? A)None of these. B)$800 ordinary loss. C)$1200 ordinary income. D)$0.

B

A client has an income tax liability of $20,000 before payments and credits. He made estimated payments of $6,000 and had $13,000 of taxes withheld from his paycheck. He also is eligible for a nonrefundable credit of $3,000 this year. Which of the following statements regarding this client's income tax situation for the current year is CORRECT? A)The client will receive a refund of $2,000 after filing his federal income tax return. B)The client will file a federal income tax return, will owe no tax, and will not receive a refund. C)The client will owe $1,000 of additional tax when he files his federal income tax return. D)The client will owe $4,000 of additional tax when he files his federal income tax return.

A

In the current year, John Smith has Schedule C net income of $25,000 from his small business, and he has a personal capital loss carry-forward of $10,000. During the current year, he sells a piece of business equipment for $5,000 that had an original cost of $10,000 and accumulated depreciation of $7,000. What is his adjusted gross income (AGI) for the current year (ignore the deduction for self-employment tax paid)? A)$24,000. B)$25,000. C)$30,000. D)$22,000.

A

Three years ago, Carol invested $10,000 in Uncle Sam Mutual Fund (the fund invests only in U.S. Treasury securities). The fund paid dividends of $700 the first year, $900 the second year, and $0 the third year (all dividends, for all years, were reinvested in the fund). The value of the fund as of January 1 of the current year is $12,000. Assuming Carol's capital gains are taxed at 15%, ordinary income is taxed at 35%, and the state tax rate is 4% for ordinary income and capital gains, what are the net after-tax proceeds that Carol will receive if she sells her shares today? A)$11,924. B)$12,000. C)$11,700. D)$11,810.

A

Joe's daughter, Alice, has been severely disabled since her birth. Alice has just turned 23 years old. Joe, a single taxpayer, provided all of her support until this year when she entered an extended care facility. The increased expenses are not covered by insurance, so Joe can only pay one-third of Alice's support, with her paternal grandmother and her aunt providing the other two-thirds, equally. Joe has come to her financial planner to understand who can list Alex on their tax return for the purposes of filing status and possible credits. What does the planner tell Joe? A) The three relatives can agree among themselves who will list Alice on their return. B)Only Joe can claim Alice on his return as he is her father. C)While one of the taxpayers may list Alice as a dependent in an agreed upon year, no tax credits would be available to that taxpayer. D)No one can list Alice on their return because no one of them has paid more than half of Alice's support.

A // Eligible taxpayers are generally free to agree among themselves who will list an individual as a dependent and who also will claim any available credits when an individual can be claimed by more than one taxpayer.

David owns a cabinet refinishing shop and bought a lathe for $15,000 to use in the business. Eight years later, David sold the lathe for $4,000 and had claimed depreciation of $5,000 on the machine. How much, if any, must David recapture as ordinary income? A)$0. B)$6,000. C)$4,000. D)$10,000.

A / /David has a realized loss on the sale of his equipment. This is calculated as follows: purchase price − depreciation claimed = adjusted basis or $15,000 − $5,000 = $10,000. Next, the sales price is deducted from the adjusted basis to give the gain(loss) realized ($10,000 − $4,000 = $6,000). Therefore, there is a Section 1231 loss on this Section 1245 property of $6,000 and no ordinary income recapture.

You are a CFP® professional and have a new client who has brought you financial information for the past 5 years. Upon review of the information you notice a discrepancy between the investment assets and the investment income reported. Your client informs you that the income from a rental property has been assigned to his adult daughter and is reported on her income tax return because she pays a lower tax rate while also providing the daughter the desired cash. The client prepares his own returns using income tax preparation software. What actions should you recommend to the client? A)Both the client and the daughter must file amended income tax returns and the client must report the rental income instead of his daughter. B)The assignment of income should be formalized by a written document prepared by an attorney. C)All the client needs to do is report the assigned income on a gift tax return because it is actually a gift from father to daughter and it will then be accounted for correctly. D)This is a routine assignment of income transaction and requires no further action on the taxpayer's part.

A //

In 2019, Jan inherited stock valued at $4,000 from her father. Her father originally purchased the stock for $500. She sold the stock for $6,000 6 months after her father's death. What gain did Jan recognize on the sale of this stock? A)$2,000. B)$6,000. C)$4,000. D)$5,500.

A // A person who inherits property receives a stepped-up basis equal to the value of the property at the decedent's death for appreciated property. Jan's basis for the sale was $4,000. Her gain on the sale was $2,000 ($6,000 − $4,000). Because inherited property is deemed to be held long-term, Jan's gain is taxed at the long-term capital gain rate.

Barry files jointly with his spouse as MFJ for federal income tax purposes and has the following gains and losses this year: --Short-term gain (STG) = $5,000. --Short-term loss (STL) = $12,000. --Long-term gain (LTG) = $10,000. --Long-term loss (LTL) = $17,000. What is the amount and character of the gain or loss that Barry will realize and recognize this year? (Realize; recognize) A)$14,000 loss; $3,000 STL. B)$14,000 loss; $14,000 LTL. C)$14,000 loss; $0. D)$14,000 loss; $3,000 LTL.

A // Barry will realize a $7,000 long-term loss and a $7,000 short-term loss, but he will get to recognize only $3,000 of the short-term loss this year. He will carry forward a $4,000 short-term capital loss and a $7,000 long-term capital loss.

Morgan, who is in the 35% marginal income tax bracket, has just sold an office building which was fully depreciated at the date of sale. The adjusted basis of the building is zero and $150,000 in depreciation taken. The land value is $30,000. Morgan's sale price was $250,000 and he is allowing the buyer to pay him using an installment sale. The buyer, Taylor, will make 20% down payment now and finance the building with annual installments over the next 10 years, with the first payment due on March 1 of next year. How much capital gain must Morgan recognize in the year of the sale? A)$44,000. B)$50,000. C)$170,000. D)$250,000.

A // Because Morgan's basis is $30,000 (the basis in the land), his gross profit percentage is 88% ($250,000 − $30,000 = $220,000; $220,000 ÷ $250,000 = 88% gross profit percentage). Each payment will consist of a 12% return of basis and 88% capital gain. Of the down payment of $50,000 ($250,000 × 20%), $44,000 is capital gain ($50,000 × 88%).

During the current year, Ginny paid the following taxes: Taxes on residence (for January 1 through August 31 of current year)$2,000State motor vehicle tax (based on the value of the automobile)$120 Ginny sold the residence on June 30 of the current year (not a leap year), and the real estate taxes were not prorated between the buyer and the seller. How much of the above taxes qualify as a deduction from Ginny's adjusted gross income (AGI) for the current year (rounded to nearest dollar)? A)$1,601. B)$1,800. C)$1,480. D)$2,000.

A // Calculation: [(180 days ÷ 243 days × $2,000) + $120] = $1,601. (Seller's days of ownership: 180 days = 31 + 28 + 31 + 30 + 31 + 29.) Real estate taxes for the entire year are apportioned between the buyer and seller on the basis of the number of days the property was held by each during the real estate tax year. The day of the sale goes to the buyer. The correct answer may be selected using an estimate of 6 of 8 months: $2,000 × 6/8 = $1,500 + $120 = $1,620 and then choosing the closest number.

Bobby owns a rental office building that he purchased for $275,000 (including $50,000 for the land). Recently, Bobby incurred the cost of a new air conditioning system for $11,300, replacement of the roof for $21,000, other miscellaneous repairs for $2,500, conversion of unused space to rental space for $16,700, cleaning services of $1,100, lawn services for $2,300, and construction to make the building handicap accessible for $23,800. Based on these expenditures, how much will be added to the cost basis of his building and will be depreciated? A)$72,800. B)$77,700. C)$51,800. D)$28,000.

A // Expenditures that materially extend the life of an asset or adapt it to a new use are considered improvements and are added to the cost basis of the asset and depreciated in accordance with the Internal Revenue Code. As a result, these expenditures include the new air conditioning system, replacement of the roof, costs to convert unused space to rental space, and the costs to make the building handicap accessible ($11,300 + $21,000 + $16,700 + $23,800 = $72,800).

When monitoring the financial news, Gary, a CFP® professional, is alerted to possible loss issues for one of his clients, Gordon. Gordon has multiple investments in shares of companies that as an industry is suffering losses in sales. Gordon has resisted Gary's recommendations to diversify his holdings in this industry. Gordon's income and his overall invested assets could be affected if the share values decrease substantially and dividend income is reduced. In one company, Gary feels Gordon's investment could become worthless in a few months. Gary's next regularly scheduled appointment with Gordon is in 4 months to review his financial situation. Regarding Gordon's circumstances, what should Gary do next? A)Gary should call Gordon and ask for an appointment to discuss the potential losses in Gordon's investments. B)Gary need not do anything because it stands to reason that Gordon will notify Gary if Gordon is adversely affected. C)Gary should make a note to Gordon's file, reminding Gary to ask Gordon during their next meeting whether the adverse financial news in Gordon's preferred industry affected Gordon's circumstances. D)Gary should assume the shares will decline in value and, for their next meeting, develop new recommendations to accommodate the change in Gordon's circumstances.

A // Gary should call Gordon and ask for an appointment to discuss the potential losses in Gordon's investments because planning may be needed to adjust the recommendations currently in place. Gary should not make any assumptions without discovering if there are, or could be, any changes in Gordon's circumstances because of the financial news. Community news is one resource for the CFP® professional to use when monitoring events that could affect a client. This financial reports and their relevance to the client's investments should trigger a request for a meeting from the planner.

Morgan, who is in the 35% marginal income tax bracket, has just sold an office building which was fully depreciated at the date of sale. Depreciation of $100,000 has been taken. The basis in the land is $100,000. Morgan's sale price was $500,000 and he is allowing the buyer to pay him using an installment sale. The buyer, Taylor, will make a 20% down payment and finance the building with annual installments over the next 10 years, with the first payment due on March 1 of next year. What is the tax treatment of the gain recognized in the year of the sale? A)Gain included in the down payment in the year of the sale is recognized at the 25% rate. B)Because Morgan is in the highest marginal income tax bracket, the capital gain is taxed at 15%. C)All gain recognized in the year of the sale is 35% gain. D)The gain is taxed at the 20% rate.

A // His gross profit percentage is 80% ($500,000 − $100,000 = $400,000; $400,000 ÷ $500,000 = 80% gross profit percentage). Each payment will consist of a 20% return of basis and 80% capital gain. Of the down payment of $100,000 ($500,000 × 20%), $80,000 is gain. Because Morgan must recapture $100,000 in straight-line depreciation taken in prior years, the $80,000 is taxed at the 25% rate for real estate depreciation recapture.

Dion is an owner in a partnership (not real estate) in which he participates during the current year. He has one partner in the business. Which of these statements does NOT indicate that Dion is a material participant in this activity for the current year? A)He participated for 450 hours during the year. B)His partner participated for 10 hours and Dion participated for 200 hours. C)Dion was a material participant for the past 6 years. D)Dion participated for 400 hours, and his partner participated for 390 hours.

A // If Dion has only participated for 450 hours during the year, he is not a material participant because he has not participated for more than 500 hours. If Dion was a material participant for the past 6 years, he will be considered a material participant for the current year because he was a material participant for 5 of the last 10 years. If Dion participated for 400 hours, and his partner participated for 390 hours, Dion would be a material participant for the current year because he has participated more than 100 hours and he has participated more than the number of hours participated by his partner. If Dion's participation is for 200 hours, and his partner participates for ten hours, Dion is considered a material participant as his participation constituted substantially all of the participation.

Meredith is a physician in Louisiana. She has owned her office building for the past 15 years. The building currently has a FMV of $400,000. Meredith has an adjusted basis in the building of $100,000. She gives the other physician $200,000 in cash in addition to her building. She decides to move her medical practice to Texas and finds a physician there who wishes to move to Louisiana. They agree to trade ownership of their respective office buildings. The value of the office building in Texas is $600,000. How much gain must Meredith recognize for income tax purposes in the year the exchange takes place? A)$0. B)$400,000. C)$500,000. D)$200,000.

A // In a Section 1031 like-kind exchange, when the boot given up is cash and not other, not-like-kind property, there is no recognized gain on the transaction. When properties are traded and the newly acquired property is used for the same purpose as the old property, the Tax Code does not require the taxpayer to recognize any gain received in the exchange.

Jenny incurred investment interest expense of $10,000 this year. The investment income she received, as a result, was $40,000, of which $5,000 was tax-free interest income on bonds. How much of her investment expenses will be deductible for this year? A)$8,750. B)$7,500. C)The amount cannot be determined from the information given. D)$10,000.

A // Interest expense generated by a source that creates tax-exempt income are not deductible. Because 12.5% ($5,000 ÷ $40,000) of her income was tax-exempt, the same proportion of her interest expense is not tax deductible. In other words, because 87.5% of her income was taxable, 87.5% of her interest expenses were tax deductible ($8,750).

Krystal is a stock broker and a CFP® professional. Her client, Maxwell, is a single tax filer who uses tax preparation software to complete his tax return each year. Maxwell has entered into a like-kind exchange during the current year. He wants assurance that all of the gain realized will be deferred on this year's income tax return when he prepares his return. Maxwell has brought all of the documentation on the like-kind exchange to Krystal's office for her review. Krystal thinks she can help Maxwell but income taxes are not her area of expertise. What should Krystal do next to help her client? A)Krystal should ask for assistance from a tax preparer or CPA to ensure Maxwell receives the correct advice. B)Krystal can advise Maxwell regarding his specific like-kind exchange transaction because a CFP® professional knows enough about tax topics to advise a client in this area. C)Because Maxwell uses tax preparation software, Krystal should advise him that the like-kind exchange will be handled appropriately by the tax preparation program. D)Krystal should tell Maxwell that like-kind treatment under Section 1031 is automatic and any realized gains will not be recognized.

A // Krystal should always strive to give her clients the best and most technically accurate advice she can. Because she is not a tax specialist, Krystal should get assistance when dealing in areas outside of her competency. Telling a client that a software program is a substitute for advice concerning tax planning is not in the best interests of the client. Without first-hand knowledge of how the software Maxwell uses to prepare his return, Krystal cannot make a general statement regarding how it will handle the like-kind exchange. Because it is possible to have recognized gain in connection with a Section 1031 exchange, Krystal's advice that like-kind treatement under Section 1031 is automatic would be incorrect.

renda, age 32, an employee of ABC Co., had an adjusted gross income of $100,000 this tax year. She incurred a variety of qualified medical expenses during the year totaling $20,000. In addition, her health insurance premiums were $2,000 for the year paid through a Section 125 plan through her employer. Brenda's health insurer reimbursed her for $18,000 of her medical bills. How much of a deduction for medical expenses would Brenda be allowed in this tax year? A)$0. B)$2,000. C)$10,000. D)$500.

A // Medical expenses are deductible only to the extent they exceed 10% of AGI in 2019, which in this case equals $10,000. The total deductible medical expenses for the year are $20,000 less any insurance recovery. Because the insurance premiums she paid were from pre-tax dollars, they are not deductible (you cannot deduct from income items which were paid for using pre-tax dollars). Brenda's total qualified medical expenses for the year are $2,000 ($20,000 − $18,000 reimbursement from the health insurer). The deductible amount is $0, because Brenda's qualified medical expenses for the year do not exceed 10% of her AGI ($10,000 in 2019). This is an increase from the 7.5% threshold in 2018.

Melinda owns a vacation home in Florida. She rents the home for 80 days per year and occupies it herself for 20 days per year. Melinda receives gross annual rental income of $15,000. Her other rental expenses total $2,000 per year. The annual real estate mortgage interest is $10,000 and the taxes cost $2,000. How much of a deduction can Melinda take on her tax return as a result of this house rental? A)$13,600. B)$14,000. C)$12,000. D)$11,200.

A // Melinda can deduct the cost of renting the home, if she occupies it for no more than 14 days per year or 10% of the number of days the property is rented. Because Melinda occupies the house for 20 days during the year and rents it out for only 80 days, this test is not met. Her gross rental income is $15,000. Because the house is used 20% of the time by Melinda (20 days out of the 100 days), she can only deduct 80% of the expenses, which equals $11,200 [80% × ($12,000 interest and taxes + $2,000 rental expenses). However, Melinda can deduct the remaining mortgage interest and real estate taxes as itemized deductions unrelated to the house rental. Her total allowed deduction is $13,600 ($12,000 interest and taxes + $1,600 other rental expenses). The remaining $400 of rental expense is classified as personal use and disallowed.

Jessica sells insurance. She uses a bedroom in her apartment as an office about 60% of the time. The square footage of the bedroom represents 15% of the overall space in the apartment. She also uses the bedroom to store personal items and, occasionally, as a bedroom for guests. Jessica's annual rental payments this calendar year were $7,500 and the annual utility costs were $1,200. How much of a deduction can Jessica take as an office expense for this tax year? A)$0. B)$567. C)$1,125. D)$675.

A // No deduction is allowed because the office is not used regularly and exclusively as a principal place of business for Jessica's insurance business, as required by the Internal Revenue Code to qualify as a deductible office expense.

Carter owns 2 life insurance policies. He has owned both policies for 10 years. Policy 1 is a $200,000 whole life policy, and Policy 2 is a $300,000 whole life policy that is considered a modified endowment contract (MEC). His cost basis in Policy 1 is $50,000, and his cost basis in Policy 2 is $200,000. Which of the following transactions would create taxable income for Carter? A)Taking a loan from Policy 2 in the amount of $100,000. B)Taking a loan from Policy 1 in the amount of $25,000. C)Receiving a dividend of $500 from Policy 1. D)Surrendering Policy 1 for $45,000.

A // Ordinarily, withdrawals from a life insurance policy receive first-in, first-out tax treatment (basis first). A loan, surrender, or dividend received on Policy 1 is nontaxable as long as it is less than Carter's cost basis. However, withdrawals from Policy 2, which is a modified endowment contract (MEC), receive last-in, first-out tax treatment (basis last). Earnings from a MEC would be withdrawn first, even for a loan.

Sara, who files MFS, filed an extension on April 15 of this year. Her AGI is well under $150,000. On June 1, she filed her tax return and owed $400 on a tax liability of $2,100. Her prior year tax liability was $2,200. Which one of the following will apply? A)Penalty and interest on $400. B)Penalty on $400. C)Failure to pay the total amount due. D)No penalty because of prepayment of over 90% of liability.

A // Sara owes penalty and interest on the $400 that she owed (and should have paid) on April 15. To avoid an underpayment penalty, taxpayers must make estimated payments that are the lesser of: (1) 90% of the tax liability shown on the current year return; or (2) 100% of tax liability shown on the prior year return if AGI is $150,000 (MFJ) or less. If AGI is greater than $150,000 (MFJ), the taxpayer must have made estimated payments of at least 110% of the prior year's tax liability.

During the year, Jack, a sole proprietor of a dog shampoo factory, had the following capital transactions: --Sold a mixing machine for $13,000. He had purchased the machine 5 years ago, and the machine had an adjusted basis of $10,000. --Sold a bottling machine that he had owned for 11 years for $42,000. The machine had an adjusted basis of $50,000. What is his net loss on this property and how will it be treated for tax purposes? A)$5,000 loss; treated as an ordinary loss. B)$8,000 loss; treated as a long-term capital loss. C)$5,000 loss; treated as a long-term capital loss. D)$8,000 loss; treated as an ordinary loss.

A // Section 1231 specifies that the net loss on the sale of all such property is treated as an ordinary loss. Therefore, Jack's total net loss is $5,000 for the two transactions [($13,000 − $10,000) + ($42,000 − $50,000)] and he is entitled to treat this loss as an ordinary loss for tax purposes.

William gave his son, Simon, a house on August 1 of this year. No gift tax was paid. The fair market value (FMV) of the house on January 1, August 1, and December 31 of the current year were as follows: $130,000, $140,000, and $150,000. William had purchased the property in 2001 for $60,000 and had used it as rental property the entire time he held it. He had taken cumulative straight-line depreciation of $10,000 through July 31 of the current year. What is Simon's initial tax basis? A)Simon's initial tax basis is the same as William's cost, less depreciation, regardless of what Simon does with the property. B)Simon's initial tax basis is the FMV on the date of the gift. C)Simon's initial tax basis is the same as William's cost. D)Simon's initial tax basis is zero because he did not pay for the house.

A // Simon's initial basis will be $50,000 ($60,000 basis less $10,000 depreciation), the donor's carryover basis. Carryover basis on a gift for which no gift tax was paid is the donor's adjusted tax basis, unless the FMV on the date of the gift is lower than the donor's basis. Basis must be reduced by any depreciation taken. If the FMV on the date of the gift is lower than the donor's basis, the donee will have a double basis. Then, the FMV of the property on the date of the gift will be the basis for losses, and the donor's basis will be the basis for gains.

Joseph, age 54 and single, earns a salary of $180,000 working for a manufacturing company. He is an avid saver, and over the years has amassed an investment portfolio of $2 million. He expects the portfolio to appreciate in value at an average rate of 8% per year. Last year he received $40,000 in dividends and interest from the portfolio. After speaking with a financial planner, Joseph decided to invest $30,000 in the current year to purchase a 15% interest in a passive activity. Operations of the activity resulted in a loss of $300,000, of which Joseph's share was $45,000. How is Joseph's loss for the current year treated for income tax purposes? A)$15,000 is suspended under the at-risk rules, and $30,000 is suspended under the passive loss rules limits. B)$45,000 is suspended under the passive loss rules limits. C)$45,000 is suspended under the at-risk rules. D)$30,000 is suspended under the at-risk rules, and $15,000 is suspended under the passive loss rules limits.

A // The at-risk rules must be applied before the passive loss rules limits. Under the at-risk rules, a taxpayer may only deduct losses to the extent of his amount at risk. Passive losses may only be deducted against passive income. Joseph has invested $30,000 in the passive activity. Therefore, his at-risk amount is $30,000. His share of the loss from the activity is $45,000, so he will be allowed to deduct only $30,000, his amount at-risk. Also, $15,000 of the loss ($45,000 less $30,000 deductible under the at-risk rules) has been suspended (disallowed) because of the at-risk rules and must be carried forward. Even though Joseph has a $30,000 loss after applying the at-risk rules, he will not be allowed a deduction for the $30,000 loss because he has no passive income. Therefore, $30,000 ($30,000 loss deductible under the at-risk rules less $0 loss deductible under the passive loss rules limits) of the loss has been suspended (disallowed) under the passive loss rules limits and must be carried forward.

Which of the following assets is NOT generally considered a capital asset? A)A computer used in a business. B)A personal residence. C)A personal auto. D)U.S. government securities held for investment.

A // The business computer is Section 1231 property, not a capital asset. A capital asset is any asset that is not a copyright or creative work, accounts or notes receivable, depreciable property used in a trade or business or for production of income (such as a computer), or inventory.

Margo files her tax return 39 days after the due date. Along with the return, she remits a check for $6,000 (the balance of the tax owed). Disregarding any interest element, her combined failure-to-file and failure-to-pay penalties are: A)$600. B)$400. C)$440. D)$660.

A // The failure-to-file penalty is netted against the failure-to-pay penalty: $60 + ($600 − $60) = $600. The failure to file penalty is 5% for each month, or part of a month. The failure to pay penalty is 1/2% per month. If both penalties apply in the same month, the maximum penalty remains 5%. The maximum aggregate penalty is 25% of the balance due.

A trust consisting of marketable stocks and bonds generated $20,000 of distributable net income (DNI) for the current taxable year. Of this amount, $5,000 is tax-exempt interest on municipal bonds. The remaining amount is made up of taxable dividends. During the year, the trustee made a $1,000 discretionary distribution of trust income to Steve, the trust beneficiary. What is the taxable portion of this distribution to Steve? A)$750. B)$250. C)$0. D)$500.

A // The taxable portion of the distribution is based on the taxable portion of the distributable net income (DNI). Therefore, $750 [$1,000 distribution × ($15,000 taxable DNI ÷ $20,000 total DNI)] of the distribution will be taxable to the beneficiary.

Fred buys a straight life annuity contract for $400,000. The contract will pay him $40,000 annually for life. Fred is expected to live 20 more years. How much of each year's annuity payment can Fred exclude from his annual gross income? A)$20,000. B)$0. C)$30,000. D)$40,000.

A // To calculate the exclusion ratio, divide the investment in the contract by the expected return on the contract. The expected return on this contract is $40,000 per year for 20 years or $800,000. Because the investment in the contract is $400,000, the exclusion ratio is 50% ($400,000 ÷ $800,000). Fred can exclude 50% of each year's payment, or $20,000, from his gross income.

Jack and Jill are in business together as partners. Their partnership agreement provides that Jack is to receive 60% of all profits and losses and Jill is to receive 40% of all profits and losses. If the partnership's total charitable contributions for the year are $10,000, how much of the charitable contribution can Jill report as a deduction on her personal income tax return? A)$4,000. B)$5,000. C)$0. D)$6,000.

A // Unless the partnership specifically describes how an item is to be distributed among the partners, it is presumed to be distributed in the same proportion as profits and losses. Because Jill is entitled to 40% of the profits and losses, she is also entitled to 40% of the charitable contributions, which equals $4,000.

Geneva has a casualty loss from a federally declared disaster of $22,100 this year. Her adjusted gross income is $100,000. How much of the loss can she deduct? A)It cannot be determined from the information given. B)$12,000. C)$2,000. D)$22,000.

B // The $100 deductible applies to the casualty loss first, resulting in a loss of $22,000. She can then only deduct the amount that exceeds 10% of her AGI, or $10,000. Her deduction is limited to $12,000.

Ken, age 23, and a full-time student at a state university, is claimed as a dependent by his parents. He earned $1,600 from a summer job this year (2019). In addition, he received $1,350 of interest income from a savings account established with funds inherited from his grandmother. He had total itemized deductions of $150 in the current year. What is Ken's taxable income this year? A)$0. B)$1,000. C)$2,950. D)$2,000.

B // $1,600 earned income + $1,350 interest income = $2,950 − $1,950 (Ken's standard deduction is limited to his earned income + $350). Therefore, taxable income is $1,000. Because Ken is claimed as a dependent by his parents, his standard deduction is reduced. The standard deduction for a dependent is limited to the greater of (1) $1,100, or (2) earned income + $350 (limited to the regular standard deduction). Because Ken's earned income is $1,600, he is entitled to claim a standard deduction of $1,950 ($1,600 + $350).

Jack and Sue sold their principal residence last month for $540,000. Their adjusted basis in the home was $280,000 and they have owned and lived in the home for 5 years. The realtor's commission was $8,400. What was their realized gain? A)$250,000. B)$ 251,600. C)$ 0. D)$260,000.

B // $251,600 is the realized gain. From the sales price of the home subtract the selling costs and the couple's adjusted basis in the home ($540,000 − $8,400 sales commission − $280,000 adjusted basis = $251,600). Jack and Sue have owned and lived in the home for 2 of the last 5 years, so they can utilize the full home sale exclusion amount for couples filing as married filing jointly of $500,000 and do not have to recognize any gain on the sale of the residence.

Assume a taxpayer is in the 35% marginal income tax bracket and has enough deductions to itemize. Calculate the equivalent tax credit that would provide the same tax benefit as a $3,000 itemized deduction. A)$1,950. B)$1,050. C)$8,571. D)$3,000.

B // A tax credit is a dollar for dollar reduction against the individual's tax liability, while a tax deduction decreases taxable income. The formula is: deduction × marginal tax rate or $3,000 × 0.35 = $1,050.

Steven, age 49, is an executive for a Fortune 500 company. He is currently taxed at the highest marginal income tax rate because of a large bonus he receives at the end of the year. He is becoming increasingly concerned with his growing tax liability each year and would like to begin investing in assets that will allow him to defer taxes. Which of the following would NOT be a good purchase for Steven? A)Ocean-front property. B)12-month CDs with daily compounding. C)Rare coins. D)Variable annuity.

B // All interest income on the CDs would be taxable in the year earned. All of the other choices will result in tax-deferred growth/appreciation and be good purchases for Steven.

Gary and Sandy have two children, ages 4 and 6. They are seeking to minimize the tax burden of the $15,000 unearned income their children are receiving this year. Which of the following strategies might help them achieve their objective? 1. Growth stocks. 2. Tax-free public interest municipal bonds. 3. Section 529 plan. 4. Family limited partnership. A)1 and 4. B)1, 2, and 3. C)1, 2, 3, and 4. D)2, 3, and 4.

B // An appropriate strategy is a Section 529 plan, which grows tax deferred and does not require income to be distributed annually to the child. The family limited partnership is not appropriate because of high setup and ongoing annual legal fees. The use of growth stock is a good strategy for sheltering minor children's income because it usually does not pay dividends (or only a very small percentage of the total return); therefore, little or no taxable income is allocated to the child. Tax-free public interest municipal bonds can also be a good strategy, as the interest earned is totally sheltered from income tax.

Marie has a paid-up whole life insurance policy with a $500,000 death benefit. Marie's financial situation is deteriorating and she needs the money prior to death to pay her current medical bills. The cash value of the policy is $250,000, but her friend, Loni, agrees to buy the policy from Marie for $300,000. Six months later, Marie dies and Loni receives the $500,000 death benefit. How much of the death benefit of $500,000 is taxable to Loni this year? A)$250,000. B)$200,000. C)$0. D)$500,000.

B // An individual who purchases life insurance from the insured in anticipation of realizing a gain when the insured dies must pay taxes on the death benefit received, less the valuable consideration paid for the policy. Loni received a $500,000 death benefit and paid $300,000 for the policy, resulting in net taxable income of $200,000. This is an example of the transfer-for-value rule.

Bob owns an apartment building which he uses for business purposes. He paid $170,000 for the building several years ago. He sold the building in the current year for $300,000. During the time he owned the building, he took total straight-line depreciation of $30,000. What is his realized gain on the sale of the building? A)$130,000. B)$160,000. C)$100,000. D)$270,000.

B // Bob's basis is his original purchase price of $170,000. His adjusted basis is the basis adjusted for the amount of cost he has recovered through depreciation, or $30,000. His adjusted basis is $140,000 ($170,000 − $30,000). He sold the building for $300,000 and when the $140,000 adjusted basis is subtracted, the net or realized gain is $160,000.

Brandon drives an SUV as his personal auto. He bought the SUV for $30,000 3 years ago, and the current FMV is $18,000. Calculate the realized and recognized gain or loss if Brandon sells his car to his friend for $18,000 today. A)Brandon will recognize and realize a loss of $0. B)Brandon will recognize a loss of $0 but will realize a loss of $12,000. C)Brandon will recognize a loss of $12,000 but will realize a loss of $0. D)Brandon will recognize and realize a loss of $12,000.

B // Brandon will realize a loss of $12,000 but will not recognize any of the loss because he cannot deduct the loss on a sale of a personal-use asset.

Harvey is a one-third partner in the HARVMAN partnership. Harvey's adjusted basis in the partnership is $20,000. Harvey and his partners have a buy-sell agreement. If Harvey dies and the $50,000 of life insurance proceeds paid to his partners are used to purchase his share of the partnership, how much ordinary income must the partnership report for tax purposes? A)$10,000. B)$0. C)$50,000. D)$40,000.

B // Buy-sell agreements allow partners to purchase a deceased partner's interest in the partnership, usually funded by purchasing life insurance on the partners. The premiums for the life insurance are not tax deductible and the proceeds received from the life insurance are not taxable as income to the partnership or to the remaining partners.

Jim has approached Cindy, his CFP® professional, for advice. His son, Bobby, a full-time college student who is age 22, inherited $500,000 from his uncle recently. The inheritance is in the form of various stocks and bonds. Bobby is overwhelmed and asked his father to help him manage the investments. Jim is not certain he can give the right advice and has asked Cindy to help him manage his son's investments. What should Cindy do? A)Cindy should ask Jim to provide her with the documents on the investments so she can advise Jim properly on how to manage the assets. B)Cindy should ask for Bobby to meet with her, bringing all the documentation on the inherited investments, so they can begin working on a financial plan for managing his investments if he chooses to become her client. C)Because Bobby will be taxed at Jim's marginal tax rate on the investment under the kiddie tax rules, Cindy must defer to Jim when working on Bobby's investments. D)Cindy should tell Jim that because Bobby is a full-time student under age 24 and Jim's dependent child, Jim must control Bobby's investments.

B // Cindy should ask for Bobby to meet with her, bringing all the documentation on the inherited investments, so they can begin working on a financial plan for managing his investments. Bobby must be Cindy's client as he is an adult, even if he is still claimed as a dependent on Jim's income tax return. Bobby may invite Jim to any meeting with Cindy, but Cindy may not divulge anything to Jim that Bobby does not approve beforehand. Cindy will be making recommendations only to Bobby, not to Jim, on the investment planning.

Eric and Graham are 50% general partners of EAG Partnership. Eric and Graham each have an adjusted basis of $100,000. Their EAG Partnership had $90,000 of income during the current year. Which of the following statements is CORRECT? A)Eric and Graham will each have an end of the year basis of $190,000. B)Graham will have self-employment income of $45,000 for the current year. C)Graham will have a new adjusted basis of $100,000 at the end of the year. D)Eric will have ordinary income of $90,000 for the current year.

B // Graham will have self-employment income of $45,000 (half of $90,000). Eric and Graham will both have a new adjusted basis of $145,000. Self Employment income, for SE tax purposes, is calculated prior to any applicable QBI deduction.

On September 19, 2018, an investor purchased 5,000 shares of Tenor Corporation for $5,000. On March 31, 2019, the stock became worthless. What is the recognized gain or loss in 2019 and how is it classified? A)$5,000 short-term capital loss. B)$3,000 long-term capital loss. C)$5,000 long-term capital loss. D)None of these.

B // If a security becomes worthless during the tax year, the loss is treated as if it occurred on the last day of the tax year. The holding period is long term. Only $3,000 may be recognized this year.

Several years ago, Barbara purchased a rental condo for $50,000. The condo now has a fair market value of $85,000, and Barbara would like to exchange it for a duplex on the north side of town. The duplex is subject to a mortgage of $25,000. Barbara will not receive any cash in the transaction. Which of the following statements is CORRECT? A)If Barbara has a net operating loss carryforward that she would like to utilize, she can elect to immediately recognize the deferred gain on the like-kind exchange in the current tax year. B)If the duplex is owned by her son, Barbara's deferred gain must be recognized if the son sells the condo within 2 years of the exchange. C)If Barbara assumes the mortgage on the duplex, she may be required to recognize gain on the like-kind exchange. D)Barbara will be subject to Section 1245 depreciation recapture at the time of the exchange.

B // If the exchange occurs with a related party, both parties will be required to recognize any deferred gain if either party disposes of their property within 2 years of the exchange. If Barbara assumes the mortgage, she will in effect be paying boot to the other party. Gain is recognized only when boot is received. Because this like-kind exchange will not be taxable to Barbara (she received no boot), she will not be required to recapture previous depreciation deductions taken. Deferral of gain on a like-kind exchange is mandatory. A taxpayer cannot elect to recognize the gain immediately.

Bradley loaned his friend, Karl, $25,000 for a down payment on a home in a zero-interest loan early in the current year. Bradley had investment income of $750 and Karl had investment income of $1,200 in the same year. The Federal interest rate is 3.5%. Karl has been making payments each month. What recommendations do you,a CFP® professional, make for accounting for the loan made to Karl by Bradley? A)Because this is a gift loan greater than $10,000 but less than or equal to $100,000, no interest will be imputed to the loan. B)Imputed interest is calculated on the loan to Karl and is considered a gift to Karl from Bradley. C)Bradley must develop an amortization schedule using the Federal rate of 3.5% to account for Karl's payments of principal and interest. D)Because Bradley's investment income is less than $1,000 this year, no interest is imputed to the loan.

B // In a gift loan, the lender makes a gift to the borrower in the amount of the imputed interest. For gift loans greater than $10,000 and less than or equal to $100,000, no interest is imputed if the borrower's net investment income for the year does not exceed $1,000. For a gift loan of more than $100,000, the prevailing federal rate of interest will be imputed. For this loan, Karl's investment income exceeds $1,000 and interest will be imputed on this gift loan.

John was involved in an automobile accident in which he was not at fault. The FMV of his car before the accident was $10,000. After the accident, the value of the car was $2,000. His insurer paid $5,000 for the loss. His adjusted gross income was $12,000 for the year. How much of a deduction can John take because of this accident? A)$1,800. B)$0. C)$1,700. D)$2,900.

B // John cannot deduct a casualty loss for the accident to his car. Only losses incurred because of a federally declared disaster are deductible casualty losses.

Which of the following statements regarding the individual alternative minimum tax (AMT) is(are) CORRECT? 1. A taxpayer must pay the AMT when the AMT calculation results in a tax liability that is greater than that resulting from the regular income tax calculation. 2. The AMT applies to any taxpayer who has losses from passive activities or other business functions. 3. The purpose of the AMT is to prevent taxpayers from using certain tax benefits to reduce their tax liability below reasonable levels. 4. The AMT reduces the tax benefits from certain types of deductions and tax preferences that are allowable for regular tax purposes. A)1, 2, 3, and 4 B)1, 3, and 4 C)1 only D)2 and 3

B // Statements 1, 3, and 4 are correct. Statement 2 is incorrect because the AMT does not apply simply because a taxpayer has losses from passive activities or other business functions.

Kathleen is a half-time sophomore at State University. Tuition payable for 2019 was $3,000. Kathleen's parents claimed her as a dependent on their 2019 income tax return and have reported an AGI of $140,000 for that year. Neither Kathleen nor her parents are claiming any other education benefit for 2019. Which of the following statements regarding the possible applicability of the American Opportunity Tax Credit is CORRECT? A)Kathleen's parents can claim $2,000 of the American Opportunity Tax Credit on their 2019 income tax return. B)Kathleen's parents can claim $2,250 of the American Opportunity Tax Credit on their 2019 income tax return. C)Kathleen can claim $3,000 of the American Opportunity Tax Credit on her 2019 income tax return. D)Neither Kathleen nor her parents can take advantage of the American Opportunity Tax Credit for income tax purposes.

B // Kathleen's parents can claim $2,250 of the American Opportunity Tax Credit on their 2019 income tax return. This amount is determined as follows: (100% × $2,000) + (25% × $1,000). Because Kathleen's parents claim Kathleen as a dependent, they are permitted to take the credit. Also, because they are currently below the applicable beginning phaseout of $160,000 (2019), there is no reduction in the amount of the credit.

for this year was $165,400, itemized deductions were $20,000, and unearned income was $25,000. What is their taxable income for this year? A)$155,000 B)$151,000. C)$165,000. D)$170,000.

B // Losses from oil and gas working interests for which the taxpayer is personally liable are deductible against active or portfolio losses without limit and without respect to the taxpayer's AGI. [$165,400 − $24,400(MFJ 2019) + $25,000 − $15,000 = $151,000]. Because itemized deduction were less than the standard deduction for a married couple, the couple will use the standard deduction.

Carol, a CFP® professional, is meeting with her client, Max. Max tells Carol he has purchased a straight life annuity and has questions on the effect the payments will have on his income taxes. Max's life expectancy is 7 years. He paid $40,000 for the annuity and it will pay him $8,000 per year. How much of the annual payment from the annuity can Carol tell Max will be excludable from his gross income? A)$5,833. B)$5,714. C)$5,901. D)$5,892.

B // Max's total investment was $40,000 and he is expected to receive ($8,000 × 7 years) or $56,000 in total. The exclusion ratio is the investment divided by the expected return ($40,000 ÷ $56,000) or 0.7143. The exclusion ratio is multiplied by the annual payment to determine the excludable amount. In this case, $8,000 × 0.7143 = $5,714.28 that is excludable, and the remaining $2,285.72 would be taxable.

Jack, Bob, and Lisa are investing in a new business together. All 3 will be material participants in the business. The business is expected to be lucrative and the substantial income will be reinvested in the business to fuel its growth. Lisa will be investing the most into the business and her stake will be larger than Jack and Bob's combined interest. Lisa has brought some concerns to her financial planner for advice. She would prefer that the business form chosen does not force her to recognize income without also providing the cash to pay any tax due on it. Which business entity should the planner recommend and why? A)Jack, Bob, and Lisa should use the S corporation form because it will allow them to control when income in recognized by the shareholders. B)A C corporation is the best fit because it will allow for the reinvestment of profits in the company and ensure that the shareholders can pay income to themselves in the form of dividends in cash, controlling how much income the shareholders recognize and when. C)A business trust with the three investors operating the business will be the best form under the trust rules. D)An LLP form will be the best fit for Lisa's needs because she can control how much profit she must recognize.

B // Only the C corporation form will allow the 3 shareholders to reinvest as much profit as needed back into the company while allowing them to control when dividends are paid to the shareholders in the form of cash. A business trust will not work because the three investors will be considered grantors and must report the trust income as it is earned, despite reinvesting it as capital in the business. An S corporation and an LLP are both flow-through entities and will require that the income be recognized whether or not it is received in cash.

Claudia, age 60, purchased an annuity this year for $25,000 that will begin paying her $1,000 each month beginning this year. Her life expectancy is 15 years. How much of the monthly payment is taxable to Claudia this year? At age 77? A)This year: $1,000; age 77: $1,000. B)This year: $861.11; age 77: $1,000.00. C)This year: $861.11; age 77: $861.11. D)This year: $138.89; age 77: $861.11.

B // Payments beyond projected life expectancy are fully taxable. (Note: if the annuity payments began on or before December 31, 1986, the exclusion ratio applies for the entire payment period). The calculation for the exclusion ratio for projected life expectancy is:1 − [$25,000 initial investment ÷ ($12,000 payments per year × 15 years)] = 0.8611 taxable percentage.$1,000 × 0.8611 = $861.11

Pete operates a printing business. In the past, the business has used the cash method of accounting. This year, Pete decided to switch to the accrual method of accounting. At the beginning of the year, the business had accounts receivable of $50,000. Assuming Pete's request for a change in accounting method is approved by the IRS, what adjustment will be required in the taxable year? A)No adjustment will be required. B)Income increase of $50,000. C)Income increase of $25,000. D)Income decrease of $50,000.

B // Pete will be required to increase his taxable income (positive adjustment) by $50,000 in the current tax year. Because Pete was on the cash method of accounting in prior years, the $50,000 of accounts receivable was never included in his taxable income (no cash was received). Therefore, when switching to the accrual method of accounting, Pete will be required to make a positive adjustment to his taxable income.

On June 12 of the current year, Bob, a single taxpayer, age 55, sold his principal residence for $400,000. He purchased the residence 4 years ago and had an adjusted tax basis of $90,000. This house has been his principal residence since he purchased it. On October 27 of the current year, he purchased a new residence for $500,000. How much should Bob recognize as a capital gain on the sale of his residence? A)$310,000. B)$60,000. C)$0. D)$110,000.

B // Single taxpayers can exclude up to $250,000 in capital gain on the sale of a primary residence, provided the residence has been the taxpayer's primary residence for 2 of the last 5 years. Use of the exclusion is limited to every 2 years for all taxpayers except under specific circumstances provided in the Tax Code.

Bob is a cash-basis, calendar year, sole proprietor. It is now December of the current year, and Bob has come to you for a strategy that would help reduce his income tax liability for the current year. You are a CFP® professional and have not seen Bob during the year. You are uncertain of his true tax position as this tax planning was not in the original letter of engagement. Which of the following actions should you perform in Bob's best interest? A)You recommend Bob buy an interest in a RELP to shelter some of his income. B)You request a new letter of engagement and updated financial information for Bob personally as well as for the business before making recommendations. C)You decide to offer basic recommendations that will help no matter what his tax status is currently. D)Because this advice was not in the original scope of work, you cannot help Bob.

B // The Practice Standards clearly require the CFP® professional to define the new scope of work in a letter of engagement and obtain updated information before making any recommendations. Offering generic recommendations may not be in the client's best interest. The planner can still help Bob if a new engagement is defined. A RELP cannot offset ordinary income, only income from another nonpublicly traded partnership.

Harry purchased 6,000 shares of IBM common stock 18 years ago for $8.75 per share. The stock split 3 times and is now valued at $75.00 per share. The broker's commission on the purchase was 3%. What is Harry's adjusted basis? A)$52,500. B)$54,075. C)$3,601,575. D)$3,600,000.

B // The adjusted basis is calculated by starting with the original purchase price (6,000 shares × $8.75 = $52,500) and increasing it by certain allowable costs. In this case, the only allowable cost that may be used to increase the basis is the broker's commission ($52,500 × 0.03 = $1,575). Harry's adjusted basis is $52,500 + $1,575 = $54,075.

Carol is a a CFP® professional who has established a new client-planner relationship with Debbie. Carol has received documentation from Debbie regarding her financial assets. Among other assets Debbie has shares of ABC Corporation, Inc. Debbie purchased the stock from ABC Corporation in 2007, for a total purchase price of $60,000. During 2013, ABC paid Debbie a tax-free distribution of $10,000. During 2016, ABC paid Debbie dividends totaling $5,000. Debbie indicated in their earlier meeting that when she considered the the distributions she had received from ABC, she was confused as to what her current basis was in the stock and has asked you for clarification. What basis amount do you determine for Debbie in 2019? A)$60,000. B)$50,000. C)$55,000. D)$45,000.

B // The basis for stocks must be adjusted by the amount of any tax-exempt distributions received. Dividends are not tax-exempt distributions and Debbie should have paid income taxes on the dividends during the year in which she received them. The dividends are not applicable in calculating the adjusted basis for the year. The tax-exempt distribution, however, must be deducted. Debbie's 2019 basis is $60,000 − $10,000 tax-free distribution, or $50,000.

Interest earned on investments is $4,000. Ralph has $16,200 of net earnings from self-employment income and paid $2,290 in self-employment taxes. He contributed $750 to a qualified retirement plan. During the year, the couple had $4,000 of unreimbursed moving expenses. What is their adjusted gross income in 2019? A)$104,306. B)$108,306. C)$105,450. D)$106,200.

B // The calculation for the Ralph and Debbie's adjusted gross income is ($32,000 Ralph's salary + $58,000 Debbie's salary + $4,000 interest + $16,200 self-employment income − $750 qualified plan contribution − $1,144 deductible of self-employment taxes) = $108,306. Qualified retirement plan contributions for a self-employed individual are deducted from gross income as an adjustment for AGI. Moving expenses are not deductible from AGI and are also not an itemized deduction. A taxpayer is allowed to deduct the employer share of the self-employment tax. In this instance it can be calculated as: $16,200 × 0.9235 × 0.0765 = $1,144.49; or = $1,144 (rounded) deductible SE tax.

Gabrielle, a CFP® professional is meeting with her client, Lisa, age 32, on June 30th of the current year for a semi-annual review of her financial plan. During this meeting Lisa discloses that she has incurred large medical bills over the past 6 months for her dependent daughter, Candy, and some of the bills were not covered by Lisa's current insurance plan through her employer. Lisa's AGI this year is projected to be $90,000. Lisa has voiced concerns about the AGI threshold hurdle for deducting Candy's medical expenses. What should Gabrielle do next to assist Lisa? A)Gabrielle should explain the 10% of AGI hurdle is waived because Lisa is has a dependent child. B)Gabrielle should request copies of the medical expenses and documentation for what has and has not been submitted to the insurer and how much Lisa is responsible to pay after reimbursement. C)Gabrielle should tell Lisa that there is nothing Lisa can do to change her medical deduction situation and just submit the documentation to her at tax preparation time. D)Because it is only mid-year, Gabrielle should encourage Lisa to defer income to 2019 to lower her AGI and allow for a higher deduction for medical expenses this year.

B // The medical expense deduction threshold for a taxpayer reverts to 10% in 2019, up from 7.5% in 2017 and 2018. Advising Lisa to defer what income she can to lower their AGI could also be useful if it is possible in her situation, but the first thing Gabrielle should do is confirm the amount of total allowable medical expenses, the amounts submitted to and paid by the insurance company, and Lisa's current unreimbursed outlay. She should not give Lisa advice until she assesses her situation regarding medical expenses. Telling Lisa that nothing can be done without first analyzing her situation violates the Principle of Diligence. Waiting until the end of the year would waste a tax planning opportunity and would not be in Lisa's best interest.

Jack is working on the final documents concerning his divorce from Lydia, which was final in 2018. He does not want to be subject to alimony recapture and is also concerned that property settlements do not adversely affect his cash flow. The bulk of the cash and property for Lydia has been identified and is to be received by Lydia as alimony over 10 years. When you review the documents you note the timeline and the amounts of alimony that has been identified by Jack. The payments in cash are $100,000 in Year 1, $75,000 in Year 2, $50,000 in Year 3 and each year thereafter for the 7 following years. What changes, if any, do you recommend to Jack? A)Because of the drop of the amount of the payments between those years is greater than $15,000, Jack will have to recapture alimony in Year 2, Year 3, and Year 4 B)There will be alimony recapture in Year 3. Jack should consider changing the amount of the payments in the timeline unless the timing of the recapture suits his tax planning strategy. C)Jack should pay the alimony in stock so Jane can decide when to realize the cash. D)Alimony can only be paid for 3 years; Jack must revise his timeline.

B // Unless there is a tax purpose for Jack or Jane in the alimony payment schedule, you should encourage Jack to consider revising the schedule to avoid including the recaptured amounts in his income in Year 3. Alimony recapture only affect the payments in the first 3 post-separation years. Jack may not use stock or any other property to pay alimony, only cash. There is no time limit on alimony payments, the schedule is decided by court decree. Divorces final on or after 1/1/2019 will not be subject to alimony recapture.

When Carl, a CFP® professional, was preparing an income tax return for George and Lois, a married couple who file jointly, he noticed some investment income items whose sources were not reflected on the statement of financial position he had prepared 6 months ago while preparing estate and retirement planning recommendations for the couple. There is $4,000 of dividend income, $1,000 of bond interest income, and a $2,000 long-term capital loss on a sale of stocks that Carl was unaware the couple owned. What should Carl do at this point in the tax return preparation? A)As long as all income items have been given to Carl, he should limit his questions to what is needed to complete the income tax return. B)Carl should ask for all documents concerning the new income items and also ascertain whether any other assets were acquired or disposed of since the last statements were compiled and in what manner (purchase, sale, inheritance, casualty loss, or gift) the assets were acquired or disposed. C)Carl should withdraw from the engagement because the clients failed to disclose all of their assets 6 months ago at the beginning of the engagement. D)Carl should ascertain whether there are any expenses associated with the income items that could reduce the taxable income reported before he finished the tax return.

B // The new income items suggest a change in circumstances for the client. The assets generating the income may have been purchased, inherited, or given to the clients as a gift. They may also have disposed of assets since the statements were produced. Carl should ask the couple for all documentation on transactions that impact their financial situation as well as interview them, asking leading questions to make certain the couple remembers to give him all the information he needs to not only do the tax return, but to continue to monitor their needs and make recommendations.

Rusty is the sole proprietor of a shoe factory. During the current year, he sold a packing machine for $2,000 that he had purchased several years ago for $10,000. The packing machine had an adjusted basis of $1,000 after depreciation. He also sold a delivery truck for $2,000 purchased several years ago which had a basis of $500 after depreciation and an original purchase price of $2,500. What is Rusty's net gain or loss on the property and how will it be treated for tax purposes? A)$1,000 gain, treated as a long-term capital gain. B)$2,500 gain, treated as Section 1245 depreciation recapture (ordinary income). C)$2,500 gain, treated as a Section 1231 long-term capital gain. D)$1,000 gain, treated as ordinary income.

B // The properties in question qualify under Section 1231, which provides special treatment for certain properties used in a trade or business (i.e., machinery and vehicles) if they are held for longer than 1 year. The properties are also subject to Section 1245 depreciation recapture. The total gain is $1,000 on the sale of the machine and $1,500 on the sale of the vehicle. The entire $2,500 is treated as ordinary income for tax purposes because of the Section 1245 depreciation recapture rules.

Ed's daughter, Gladys, transferred the ownership of her house to Ed 7 years ago as a gift. Gladys originally paid $50,000 for the house 15 years earlier. When she gifted the house to Ed, it was valued at $75,000. Ed died this year and left everything he owned to Gladys, including the house she originally gifted to him. At Ed's death in 2019, the house had a fair market value of $100,000. Gladys sold the house at the end of the same year for $150,000. What is Gladys' gain on the sale of the house for income tax purposes? A)$50,000 less depreciation. B)$50,000. C)$150,000. D)$100,000.

B // The step-up provisions do not apply to property that is inherited by the person who gave it to the decedent as a gift within one year of the decedent's death. Because Ed was given the property well over one year prior to his death, the property has a stepped-up basis when it is inherited by Gladys. Her new basis on the property will be the FMV at Ed's death, or $100,000. After selling the property for $150,000, she realized a gain of $50,000.

When is a corporation most likely to elect S corporation status? A)The corporation places personal liability on the shareholders. B)A loss is anticipated especially at the start-up of a business. C)The corporation desires to issue preferred stock. D)The corporation desires to borrow money from shareholders.

B // The taxable income or losses of an S corporation flow through to the shareholders. Therefore, losses can be utilized by the shareholders on their individual tax returns to reduce their own liability. Although a corporation may borrow money from shareholders, this is not exclusive to S corporations and is not a valid reason to select such a corporate-status.

Craig gives his son Scott stock with a basis of $80,000 and a fair market value of $70,000. No gift tax is paid. Scott subsequently sells the stock for $78,000. What is Scott's recognized gain or loss? A)$8,000 gain. B)No gain or loss. C)$78,000 gain. D)$2,000 loss.

B // This is a gift of loss property and the double basis rule applies. Scott's gain basis is $80,000. His loss basis is $70,000. Because his selling price of $78,000 is between the gain basis and the loss basis, he has no recognized gain or loss.

Billy has established a trust that pays out $2,000 each month to his mother, Wendy. The trust department of Allword Bank acts as trustee. Billy retains the right to borrow assets from the trust without security, and he is the remainder beneficiary of the trust. Who must pay the income tax on the $40,000 earned by the trust and in what amount? A)The trust pays on $24,000; Billy pays on $16,000. B)Billy pays on $40,000. C)Wendy pays on $24,000; Billy pays on $16,000. D)Wendy pays on $24,000; the trust pays on $16,000.

B // This is a grantor trust; therefore, the grantor (Billy) will be responsible for the tax on all of the income. Billy's right to borrow trust assets without security will cause the trust to be a grantor trust for income tax purposes. The distribution to his mother is a gift from Billy.

Tommy inherited 200 shares of ACME, Inc., stock from his uncle, who died 3 months ago. His uncle originally bought the stock for $5,300. The value of the stock was $6,000 on the date of his uncle's death. Tommy sold the stock when its value rose to $6,500. Tommy's taxable gain on the sale is: A)$1,200 long-term capital gain. B)$500 long-term capital gain. C)$1,200 short-term capital gain. D)$500 short-term capital gain.

B // When Tommy's uncle died, the stock received a step-up in basis to the fair market value as of the date of death. Therefore, Tommy's basis in the stock is $6,000. Tommy sold the stock for $6,500, so his taxable gain is $500 ($6,500 sales price less $6,000 basis). All inherited property is considered long-term to the heirs of the estate.

Steve began his professional corporation single practitioner CPA firm 38 years ago. He worked profitably as a sole practitioner for the full 38 years until retiring December 31 of last year at his full retirement age. On January 1 of this year, he sold his practice for $400,000 to be received in 4 equal, annual payments of $100,000, beginning on the date of sale and continuing each January 1 for the next 3 years. Is Steve eligible for Social Security retirement benefits during this year and why? A)No, even though he retired at his full retirement age, the proceeds from the sale of his practice will delay his receiving Social Security retirement benefits. B)Yes, he is eligible because he retired at his full retirement age and is fully insured. C)No, he is not considered retired for Social Security purposes until the installment payments are complete. D)Yes, he is entitled to benefits, but they will be reduced because of the payments from the sale of his practice.

B// Social Security benefits may be reduced for individuals under full retirement age with earned income above certain levels. The proceeds of the installment sale are not considered earned income for Social Security retirement benefit purposes. Therefore, Steve's benefits will not be reduced. He has attained his full retirement age and is fully insured. Note: The reduction of Social Security benefits based on earned income no longer applies once an individual reaches full retirement age. Because Steve retired at his full retirement age, his benefits would not be reduced, even if he had earned income.

Polly bought a new machine at a cost of $12,000 for her business a few years ago and immediately placed it into service. She claimed total depreciation deductions of $6,000 in previous tax years. She sold the machine for $2,000 in the current year. How much of a loss can Polly claim? A)$6,000. B)$4,000. C)There is no loss associated with this sale. D)$2,000.

BB //If the adjusted basis of the disposed property is greater than the amount realized, the difference is a loss. In this case, the adjusted basis is $12,000 − 6,000 = $6,000. She sold the equipment for $2,000; therefore Polly's loss is $6,000 − 2,000 = $4,000.

Ken, age 23, a full-time student at a state university, is claimed as a dependent by his parents. He earned $1,600 from a summer job this year (2019). In addition, he received $1,350 of interest income from a savings account established with funds inherited from his grandmother. He had total itemized deductions of $150 in the current year. What is Ken's taxable income this year? A)$2,000. B)$0. C)$1,000. D)$2,950.

C // $1,600 earned income + $1,350 interest income = $2,950 − $1,950 (the standard deduction is limited to earned income + $350). Therefore, taxable income is $1,000. The standard deduction for a dependent is limited to the greater of (1) $1,100, or (2) earned income plus $350 (limited to the regular standard deduction). Because Ken's earned income is $1,600, he is entitled to claim a standard deduction of $1,950 ($1,600 + $350).

Alberto Corporation, a C corporation, distributes to its only 2 shareholders a piece of real estate located in the Bahamas. Alberto Corporation's basis in the property is $40,000. The property's FMV when distributed is $100,000. Alberto's current and accumulated earnings and profits for the year are $100,000. The 2 shareholders will have to pay the year's property taxes on the real estate, which will equal $10,000. If the shareholders split all distributions and liabilities equally, how much dividend income must each shareholder report? A)$20,000. B)$0. C)$45,000. D)$50,000.

C // A distribution of property is treated as a dividend that equals the FMV of the property. The distribution of a $100,000 piece of real estate is treated entirely as a dividend because that amount does not exceed current and accumulated earnings and profits. Distributions are reduced by the amount of any immediate liability that accompanies the ownership of the property, such as the property taxes. The dividend amount is reduced by $10,000 for a total of $90,000. Each shareholder must report one-half of that amount, or $45,000, as dividend income.

Bob and his brother, George, are interested in forming a business together. However, they are concerned that the business will have losses for the next 3 years, whether the business would continue if one brother died, and whether they will have some limited liability protection. On the basis of the concerns of the brothers, which form of business entity is the most appropriate? A)C corporation. B)Partnership. C)S corporation. D)Limited partnership.

C // An S corporation will allow losses to flow through to the brothers, will continue if one brother dies, and will have the liability protection of a C corporation.

You are a CFP® professional and are meeting with your client Brenda to monitor her ongoing financial status. Brenda owns a vacation home in New Mexico. She rented the property to others for the entire year, except for 10 days during the summer when she and her family used it for their vacation. The gross rental income that Brenda received is $65,000. Her rental expenses total $5,000. Brenda would like you to explain how this will change her income tax situation, in particular, how much of the rental expenses are deductible. After reviewing the documents she sent to you prior to the meeting, you have an answer for Brenda. How much of a deduction for rental expenses can Brenda take on her tax return? A)$4,721. B)$3,411. C)$4,863. D)$5,000.

C // Brenda can deduct the cost of renting the home if she occupies it for the greater of no more than 14 days per year or for 10% of the number of days the property is rented. Because Brenda occupied the house for only 10 days during the year, this test is satisfied. Even though this rental use exception is allowed, the deductible expenses related to the rental of the house are limited. Specifically, she can only deduct a portion of the actual rental expenses, which equals the number of days during the year that the house is rented to others, divided by the total number of days that the house is used by either tenants or Brenda. Given 365 days per year, Brenda's tenants occupy the house all but 10 days, for a total of 355 days. She is allowed to deduct 97.26% (355 days ÷ 365 days) of the $5,000 rental expenses, which equals $4,863.

Barbara, a CFP® professional, is meeting with her client Jerry at the end of the tax year. Jerry reports he has received dividend income from Brando Manufacturing, Inc. this year. From the records Barbara has on Brando for this tax year, she learns the following: --Jerry is the sole shareholder of Brando Manufacturing, Inc., a C corporation. --His basis in the stock is $10,000. --This year, Brando had current and accumulated earnings and profits of $97,000, but distributed $111,000 to Jerry. Jerry believes this is all dividend income. How much of this distribution does Barbara tell Jerry represents a capital gain to Jerry and how much represents dividend income? A)$14,000 capital gain; $97,000 dividend income. B)$0 capital gain; $111,000 dividend income. C)$97,000 dividend income; $4,000 capital gain; $10,000 return of capital. D)$0 capital gain; $97,000 dividend income.

C // Dividends are taxable to the recipient as dividend income, but only to the extent of the corporation's current and accumulated earnings and profits. Because Brando had $97,000 in earnings and profits, the first $97,000 of the distribution would be taxable as dividend income to Jerry. The next $10,000 of the distribution would be treated as a tax-free return of Jerry's basis, and the remaining $4,000 is a capital gain.

Brad is the sole shareholder in Parker Inc., a C corporation. His basis in the stock is $10,000. This year, Parker had current and accumulated earnings and profits of $50,000. Parker distributed $70,000 to Brad this year. How much dividend income must Brad report on this year's income tax return? A)$70,000. B)$60,000. C)$50,000. D)$40,000.

C // Dividends are taxable to the recipient as dividend income, but only to the extent of the corporation's current and accumulated earnings and profits. Parker had earnings and profits of $50,000, so only $50,000 of the distribution is dividend income to Brad.

Boyd, a shareholder in the Pivitz Corporation, owns 50% of the shares of stock in the company. This year, Pivitz had current and accumulated earnings and profits of $50,000. Pivitz paid its shareholders a cash dividend of $50,000, of which Boyd is to receive 50%. How much dividend income must Boyd report on his income tax return? A)$25,000, less his basis in the stock. B)$0. C)$25,000. D)$50,000.

C // Dividends are taxable to the recipient as dividend income, but only to the extent of the corporation's current and accumulated earnings and profits. Pivitz had earnings and profits of $50,000 and paid a dividend of $50,000, of which the entire dividend is taxable as dividend income to the recipients. If Boyd received half of that amount, he realized $25,000 of taxable dividend income.

Frances is a family practice doctor who operates as a sole proprietor. She bought an X-ray machine several years ago for $50,000. She used the MACRS 7-year percentages to recover the cost of the machine. Frances sold the machine in the current year for $58,000 and plans to replace it with an MRI machine. She has claimed $14,800 of cost recovery deductions. How much will her Section 1245 recapture be this year? A)$22,800. B)$35,200. C)$14,800. D)$0.

C // Frances calculates her Section 1245 recapture as follows: purchase price − depreciation claimed = adjusted basis or $50,000 − 14,800 = $35,200. Next, the adjusted basis is deducted from the amount realized to determine the gain/(loss) realized: $58,000 − 35,200 = $22,800 There is a gain realized and recognized of $22,800. Her Section 1245 recapture recognized as ordinary income is the lesser of the gain realized or cost recovery deductions claimed ($22,800 vs. $14,800). In this case, the Section 1245 recapture as ordinary income is $14,800. There is also a long-term capital gain of $8,000.

Maxwell established an irrevocable trust for his son, Jeff, whom he is legally obligated to support. The trust income is used to support Jeff, and the trustee is free to use any portion of the income for that purpose. In the current year, 25% of the income is used for Jeff's support. Based on this information, which of the following statements is CORRECT? A)The trust will pay taxes on 100% of the trust income. B)Maxwell will pay taxes on 100% of the trust income. C)Maxwell will pay taxes on 25% of the trust income. D)The trust will pay taxes on 25% of the trust income.

C // In this case, the income is used to fulfill Maxwell's support obligation and is, therefore, benefiting the grantor. The grantor of a trust is taxed on the income from the trust when the income is used for the grantor's benefit. However, only the portion used for the grantor's benefit is taxable. In this situation, Maxwell will pay tax on 25% of the income, while the trust will pay tax on 75% of the income.

James has gross income of $40,000 this year. He pays $3,000 per year in property taxes on his home. He also pays property taxes of $12,000 on an apartment building that he owns. Assuming no other adjustments to gross income, how much is James' adjusted gross income (AGI) for the year? A)$37,000. B)$40,000. C)$28,000. D)$30,000.

C // Most individual tax payments are deductible only as itemized deductions from AGI limited to an aggregate total of $10,000. If the taxpayer does not have enough deductions to itemize, there is no tax benefit from the payment of state, local and even foreign taxes paid on income or property. In this example, James would derive greater benefit by taking the standard deduction. Thus he will not claim the $3,000 in personal taxes as an itemized deduction. However there are two cases in which the taxpayer may deduct taxes from gross income in arriving at the adjusted gross income: when the taxes are incurred in carrying on a trade or business. when the taxes are incurred in connection with property held for the production of rents or royalties. Taxes paid on the apartment building can be deducted from James's gross income to arrive at his AGI. Because James is carrying on a trade or business, the $12,000 of taxes on the apartment building are deductible from AGI. Therefore, James' AGI is $28,000 ($40,000 − $12,000).

Benny and Henry are equal partners. During the first year of operation, the partnership suffered a loss of $50,000. Benny's adjusted basis in the partnership is $20,000. How much of the $50,000 loss can Benny claim on his personal income tax return for the year the loss occurred? A)$30,000. B)$25,000. C)$20,000. D)$0.

C // Partners can claim their share of the partnership's loss, but not beyond their adjusted basis in the partnership. Benny's one-half share of the $50,000 loss equals $25,000, which exceeds his adjusted basis of $20,000. He is entitled to claim only a $20,000 loss on his tax return, and the remaining $5,000 loss may be carried over to a future year when Benny has an increase in his basis in the partnership through either earnings or investment.

Thomas is considering an investment in a real estate limited partnership (RELP). He plans to contribute $20,000 to the partnership, which is raising capital for the development of a boat storage facility. Assuming he makes this investment, and his adjusted gross income was $160,000, which of the following statements is CORRECT? A)If the partnership has an operating loss for the year, Thomas can use his allocable share of the loss to offset his interest and dividend income. B)If the partnership has an operating loss for the year, Thomas can use his allocable share of the loss to offset any capital gains from the sale of his stocks. C)If the partnership has an operating loss for the year, Thomas cannot deduct his allocable share of the loss, unless he has passive income from another activity. D)If the partnership has operating income for the year, the income will be taxed to the partnership, unless a cash distribution is made to the partners.

C // Passive losses can only offset passive income. Passive losses cannot offset investment gains or income. Partnerships pass through their income to the partners, who report their share on their tax return.

MSC, Inc. is a closely held C corporation that manufactures boilers. The company has been in business for over 45 years. MSC has active income this year of $250,000 and no passive or portfolio income. The company also leases equipment that generates passive losses of $120,000 per year. How much of the passive loss can the company use this year? A)$0. B)$100,000. C)$120,000. D)$25,000.

C // The company can deduct the entire $120,000 because it is a closely-held C corporation that is not a personal service corporation. Passive losses may be used to offset active, but not portfolio income.

When reviewing the documents provided by your client, Randy, you, a CFP® professional, note that Randy implemented the plan to acquire new stores and expand his business, in accordance with his stated financial goals. Randy, a calendar-year, cash-basis taxpayer, owns and operates furniture rental outlets in Georgia. He wants to expand to other states. He spent $20,000 investigating furniture stores in Alabama and $12,000 investigating stores in Florida. He acquired the Alabama stores but not the stores in Florida. Randy is understandably concerned about the expenses he incurred investigating the stores in both states. He asks you to review his expenditures and tell him regarding the income tax implications. When reporting the above expenses on his income tax return you tell Randy he should: A)capitalize $20,000 and not deduct $12,000. B)capitalize $32,000. C)expense $32,000. D)expense $12,000 and capitalize $20,000.

C // Randy is already in the business; therefore, expenses are currently deductible and do not need to be capitalized. Deductible expenses include travel, engineering and architectural surveys, marketing reports, and legal and accounting services.

Delbert is a single taxpayer and is an active participant in rental real estate. He has earned income of $75,000, $5,000 of unearned income, $12,500 in deductions, and $5,000 in losses related to his real estate holdings. What is Delbert's taxable income? A)$54,500. B)None of these. C)$62,500. D)$69,500.

C // Rental and real estate passive losses are allowed up to $25,000 ($12,500 for MFS), which can be used to offset nonpassive income. Delbert has income of $80,000 less deductions and allowable losses of $17,500 ($5,000 + $12,500), leaving a taxable income of $62,500.

Martha owns stock for which she paid $10,000 at the time of purchase 3 years ago. She is considering selling the stock this month for a gain of $5,000. This year, she has also sold her personal residence which she has owned and lived in for 10 years for a gain of $75,000 over the purchase price. She is concerned about her current tax situation and the fact she has $80,000 of realized income in addition to her salary of $100,000 in the same tax year. Martha has consulted you, a CFP® professional, for advice. Which of the following are recommendations you should make to Martha? 1. Before the tax year ends Martha should pay the estimated taxes on the $80,000 of additional income. 2. Martha should review her tax deposits for the year to insure she will not be assessed a penalty for underpayment. 3. If the stock sold was Section 1244 stock, the gain does not receive special treatment for income tax purposes. 4. Martha will not have to recognize all of the combined gain she realized on the sale of her stock and the personal residence and plan her tax deposits accordingly. A)1 and 3. B)1 and 2. C)2 and 4 D)1, 2, 3, and 4.

C // Statements 2 and 4 are correct. Realization of income may occur on a specific transaction, but it may not have to be recognized (reported for tax purposes) at the same time. The $75,000 gain on the sale of her home is Section 121 gain and is excluded from her income because she owned and lived in the home for 10 years (ownership and use tests). Section 1244 stock rules apply to losses on the sale of the Section 1244 stock, not gains. However, any gain on the sale of the Section 1244 stock is long-term capital gain and taxed at the special long-term capital gain rate. Martha will only recognize the gain on the sale of the stock, not the residence sale gain.

Jim, recently retired, is 62 years old and expects to be in the maximum federal and state tax bracket. He plans to liquidate three of the investments listed below. Assume that each investment is worth $50,000 and has grown from an original investment of $25,000 over a period of more than 3 years. Liquidation of which three of the following investments, in order of priority, would result in the lowest tax liability for Jim? (CFP® Certification Examination, released 08/04) 1. 401(k) plan. 2. CD paying 4%. 3. U.S. savings bonds. 4. Traditional IRA (contributions were not deductible). 5. A blue chip stock. A)2, 5, 4. B)2, 3, 1. C)2, 5, 3. D)3, 5, 1.

C // The CD, blue-chip stock, and savings bonds (in that order) would result in the lowest tax liability upon liquidation. Because the interest on the CD was taxable each year, the basis in the CD would be equal to the value. Therefore, no federal or state income taxes would result upon liquidation. The appreciation of the blue-chip stock would be taxed, but at favorable capital gains rates. For U.S. savings bonds, the appreciation would be taxed at ordinary income tax rates for federal income tax purposes. However, savings bonds are not subject to state income taxes. For the traditional IRA, the appreciation only (client has basis for the contributions because they were nondeductible) would be taxed at both federal and state ordinary income tax rates. For the 401(k) plan, the entire amount would be taxed at both federal and state ordinary income tax rates.

Jason, who is in the 35% marginal income tax bracket, wants to sell an office building which was fully depreciated at the date of sale (the basis in the land is $50,000). His brother, Sam, referred him to Sam's own CFP® professional to make certain Jason knows the tax effects of the sale. Jason tells the CFP® professional his adjusted basis in the building is zero and $150,000 in depreciation has been taken. Jason's sale price was $500,000 and he is allowing the buyer to pay him using an installment sale. The buyer, Taylor, will make 20% down payment and finance the purchase with annual installments over the next 10 years, with the first payment due on March 1 of next year. Jason engages the CFP® professional to advise him on the transaction. What should the CFP® professional do next regarding the sale transaction, assuming Jason becomes a financial planning client? A)The CFP® professional should advise Jason on the tax ramifications based on Jason's verbal recitation of the facts. B)Because Sam is the CFP® professional's client, the CFP® professional cannot help Jason. C)The CFP® professional should ask for the documents supporting the numbers Jason has provided. D)Jason would have to also bring in Taylor to see the CFP® professional because he would have to help both sides in the transaction.

C // The CFP® professional would need to request all of the supporting documentation for the proposed sale before advising what the tax ramifications may be. He cannot rely on only the client's verbal recitation of the facts.

Which property listed below, if sold in 2019, would produce the greatest amount of cash received by the seller after taxes? Assume the property was held for 2 years, was sold for $10,000, and had basis of $1,000.(For the purposes of this question, disregard the taxpayer's AGI and the possible effect of the 3.8% net investment income surtax.) A)All of these would result in the same after-tax cash amount. B)Commercial building (fully depreciated). C)IBM stock. D)Baseball card collection.

C // The IBM stock would result in the greatest amount of cash received because it is taxed at a maximum 0%/15%/20% capital gain tax rate, depending on the taxpayer's AGI. The commercial building would have an up to 25% rate. The baseball cards are taxed at an up to 28% because they are a collectible.

Daniel gave Mike securities in the current year. Daniel's adjusted basis for the securities is $48,000, and the fair market value on the date of the gift was $40,000. Gift tax of $2,000 was paid by Daniel. What is Mike's basis for the stock for gain and for loss? A)$50,000 for gain; $42,000 for loss. B)$40,000 for gain; $40,000 for loss. C)$48,000 for gain; $40,000 for loss. D)$0 for gain; $0 for loss.

C // The donee's gain basis for the property received is the same as that of the donor. The donee's loss basis is the lower of the donor's adjusted basis or fair market value on the date of the gift. Basis will be increased for the portion of the gift tax that is attributable to the donor's unrealized appreciation. In this example, there wasn't any unrealized appreciation so there wasn't an increase in basis.

Your client, Philip, is considering investing in bonds. He is trying to decide between a corporate bond paying a 7% coupon and a municipal bond paying 5.3%. Philip's marginal federal income tax rate is 32%, and he lives in a state that does not impose an income tax. If Philip's objective is to maximize the after-tax income from his bond investment, which bond would you recommend that Philip purchase? A)Both bonds will provide the same after tax yield because Philip is not subject to state income tax B)The municipal bond because its after tax yield is 7.68% C)The municipal bond because its after tax yield is 5.3% D)The corporate bond because its after tax yield is 10.15%

C // The interest from the corporate bond is subject to federal income tax, while the interest from the municipal bond is exempt from federal income tax. Therefore, the after tax yield of the municipal bond is 5.3% (same as the coupon rate) and the after tax yield of the corporate bond is 4.76% [7% x (1 - .32) = 4.76%].

Bobby, a single taxpayer, purchased a 2,500-square-foot home for $260,000. Six months later, he received a promotion at work which required him to relocate. He sold the home for $220,000. What are the tax ramifications of the sale, assuming Bobby had no other property transactions? A)Bobby can take a $40,000 ordinary loss deduction this year. B)Because the promotion forced him to sell the home, Bobby is entitled to a partial loss deduction of $31,250. C)Bobby has a realized loss of $40,000, but not a recognized loss. D)Bobby can deduct $3,000 as a capital loss, and will have a $37,000 capital loss carryforward.

C // The loss on the sale of a personal-use asset (i.e., personal residence or personal auto) is nondeductible. Bobby has realized a loss of $40,000 but can not recognize that loss for income tax purposes.

Brittany, age 17, earns $5,000 in salary. Kate, age 7, has $2,300 in dividends from a mutual fund. Tony, age 3, has $1,100 in interest from a savings account. Amanda, age 19 and not a full-time student, has $2,000 in dividends and interest. Which of the following children has(have) income subject to federal income tax at estates and trusts tax rate in 2019? A)Kate and Tony. B)Brittany. C)Kate. D)Kate, Tony, and Amanda.

C // This question is related to the kiddie tax, which applies to unearned income in excess of $2,200 (for 2019), by a child under the age of 19 or a full-time student under age 24. Although Brittany is younger than 19, she has only earned income. Amanda, age 19, is not subject to kiddie tax because her unearned income does not exceed $2,200. Tony, age 3, had unearned income less than $2,200. Therefore, the correct answer is Kate, who at age 7, had unearned income of $2,300.

Mark has approached you for advice on structuring when he receives taxable income and how the constructive receipt rules may affect his business and personal income. He uses the accrual basis of accounting in his business, which is an LLC taxed as a corporation. He recently sold his personal antique decoy duck collection to a friend at a significant gain. It is near year end and his friend has offered Mark a check for the sales price. What do you recommend to Mark? A)Mark can tell his friend to hold on to the check until he is in a better position to minimize the tax impact on his personal income next year. B)Because Mark's business is on the accrual basis, he must recognize any income he receives personally on the accrual basis. C)Mark must include the gain on the sale of the collection in his income this year as the check is available to him now. D)The income on the collection sale may be deferred to the next tax year as it has not actually been received by Mark this year.

C // Under the constructive receipt rules, income for which there is no substantial barrier to the cash basis taxpayer to take control of is considered to have been constructively received by the taxpayer in the year it became available. The check for sale of the collection has been written and is available to Mark. The method of accounting of Mark's corporation does not affect Mark's personal method as a cash basis taxpayer.

Scott received an incentive stock option (ISO) from his employer. The option allows Scott to purchase 100 shares of company stock at $50 per share. Scott exercised the option 5 years later when the fair market value (FMV) of the stock was $125 per share. He held the stock for 3 more years and sold it on the secondary market for $175 per share. In the year of exercise, Scott had reportable amounts for regular tax purposes and for alternative minimum tax (AMT) purposes, respectively, of: A)$7,500 and $5,000. B)$0 and $0. C)$0 and $7,500. D)$12,500 and $5,000.

C // When an incentive stock option is exercised, the bargain element (the difference between the FMV and the option price) on the date of exercise is an AMT preference item. On exercise, the bargain element is $7,500 [($125 − $50) × 100 shares]. The exercise of an incentive stock option has no effect on regular taxable income.

Jenny is a real estate agent and investor in Georgia. She holds an office building as one of her real estate properties she is offering for sale, that has a FMV of $200,000, which she is trying to sell. She has an adjusted basis in the building of $100,000. Her brother, a doctor, owns an office building in California and agrees to trade buildings with her. The value of the office building in California is $300,000. If the exchange takes place this year, how much of a gain must Jenny recognize this year because of the exchange? A)$0. B)$300,000. C)$200,000. D)$100,000.

C // When properties are traded and the use for the newly acquired property is the same as for the old property, the Code does not require the taxpayer to recognize any gain received in the exchange. However, the property that is traded cannot be treated this way if it is property held primarily for sale. Dealers in real estate who hold properties that are primarily for sale, such as the case with Jenny, must recognize any gain or loss of an exchange. The gain in this case is the value of the building received less Jenny's basis in her building. ($300,000 − $100,000 = $200,000 gain)

ACME Corporation had accumulated earnings of $400,000 after the end of Year 1. In Year 2, ACME made a profit of $10,000. ACME distributed $200,000 to its shareholders at the end of Year 2. What is ACME's accumulated earnings figure for Year 2 tax purposes? A)$400,000. B)$410,000. C)$210,000. D)$200,000.

C // When the corporation makes a distribution, its earnings and profits are reduced by that amount. Total earnings and profits for Year 2 were $410,000 and the distribution was $200,000. The accumulated earnings for Year 2 were $210,000.

John is the sole shareholder of River Rafting, Inc., a C corporation. In the current year, he receives a salary of $150,000 and dividends of $60,000 from the corporation. River Rafting's taxable income for the current year is $400,000. On an audit of River Rafting, the IRS reduces his salary by $60,000 to $90,000 because it was determined to be unreasonable. Which of the following statements regarding the IRS's action is CORRECT? A)John's gross income will increase by $60,000 as a result of the IRS adjustment. B)John's gross income will decrease by $60,000 as a result of the IRS adjustment. C)River Rafting's taxable income will increase by $60,000 as a result of the IRS adjustment. D)River Rafting's taxable income will not be affected by the IRS adjustment.

C //The $60,000 of salary is reclassified as a dividend. Thus, River Rafting's taxable income increases by $60,000 because dividends are not deductible. John's gross income remains the same. His salary decreases by $60,000, but his dividend income increases by $60,000.

On February 2 of the current year, a taxpayer exchanged a bank building, having an adjusted tax basis of $600,000 and subject to a mortgage of $275,000 for another bank building with a fair market value of $800,000 and subject to a mortgage of $275,000. Transfers were made subject to outstanding mortgages. What amount of gain should the taxpayer recognize in the current year? A)$200,000. B)$75,000. C)$275,000. D)$0.

D

Carter, an unmarried individual, has an adjusted gross income (AGI) of $180,000 in the current year before any IRA deduction, taxable Social Security benefits, or passive activity losses. Carter has incurred a loss of $30,000 from rental real estate in which he actively participated. What amount of the loss attributable to this rental real estate may be used in the current year as an offset against income from nonpassive sources? A)$12,500. B)$30,000. C)$25,000. D)$0.

D // An exception to passive loss limits regarding real estate allows individuals to deduct up to $25,000 of losses from real estate activities against active and portfolio income. However, the annual $25,000 deduction is reduced by 50% of the taxpayer's AGI in excess of $100,000. The deduction is entirely phased out at an AGI of $150,000.

On January 1, Dennis loaned his daughter, Betty, $90,000 to purchase a new personal residence. There were no other loans outstanding between Dennis and Betty. Betty's only income for the year was $30,000 in salary and $4,000 in interest income. Dennis had investment income of $200,000. Dennis did not charge Betty any interest on this loan. If the relevant Federal rate is 5% for the current year, how should you recommend Dennis and/or Betty treat the interest income or expense for this loan this year? A)Betty must recognize $4,500 (0.05 × $90,000) imputed interest income on the loan. B)Dennis must recognize imputed interest income of $4,500. C)Betty is allowed a deduction for imputed interest of $4,500. D)Dennis must recognize imputed interest income of $4,000.

D // Because the loan is not more than $100,000, the imputed interest will be calculated using the applicable federal rate less the actual rate, limited to Betty's net investment income. Thus, Betty's imputed interest income after applying the federal rate would be $4,500 (5% of $90,000). Dennis would impute the lesser of the federal rate or Betty's net investment income. Because Betty's net investment income is only $4,000, the imputed interest income to Dennis is $4,000, and the imputed interest expense to Betty is $4,000. Dennis has also made a gift to Betty of $4,000, which may be offset by the gift tax annual exclusion if he has made no other gifts to her during the year totaling more than $15,000 (for 2019).

Scott just sold a building for $180,000 that had a basis of $80,000. The property had suspended passive losses of $50,000. Scott manages a portfolio of private activity bonds that paid interest of $25,000. As a high school physics teacher, Scott earns $25,000, of which he contributes $10,000 to his tax-sheltered annuity (TSA). What is Scott's AGI? A)$15,000. B)$40,000. C)$115,000. D)$65,000.

D // Capital gain of $100,000 is included in AGI (although it may be taxed at a preferential rate). The passive loss of $50,000 is allowed because of the disposition of the property. Municipal bond interest is not included in AGI, although it could be AMT issue. Scott's salary of $25,000 is included in AGI. The TSA contribution of $10,000 reduces current income because TSAs are tax-deferred retirement savings plans. $100,000 − $50,000 + $25,000 − $10,000 = $65,000.

Mary has had a difficult year financially. She suffered a loss in her business when some merchandise failed to sell. The goods were sold to a discounter at $19,000 below her cost. Mary had land she had invested in 5 years ago at a cost of $100,000. The county opened a trash dump 300 feet from her property. She sold the land this year for $65,000, suffering a 35,000 loss on the sale. She has asked you, a CFP® professional, to tell her how to classify her losses for tax purposes. Which of the following is a difference between an ordinary loss and a capital loss? A)You should tell Mary ordinary losses are deductible, but capital losses are not deductible. B)You should tell Mary ordinary losses are limited with respect to the amount that can be deducted from income, while capital losses are not subject to such limitations. C)You should tell Mary ordinary losses are not deductible, but capital losses are fully deductible. D)You should tell Mary ordinary losses do not result from the sale or exchange of a capital asset, unlike capital losses.

D // Capital losses result from the sale or exchange of a capital asset. Ordinary losses result from something other than the sale or exchange of a capital asset. Capital loss deductions, not ordinary loss deductions, are subject to limitations.

After 5 years of marriage, Beth and Rudy, who have 2 children, filed for divorce. After some hastily prepared legal proceedings on both sides, the court granted the divorce on 12/30/18 and ordered Rudy to pay $1,000 per month in alimony and $1,500 per month in child support. How much is Beth required to include in gross income for each year she receives these payments? A)$18,000. B)$28,000. C)$30,000. D)$12,000.

D // Payments of alimony or separate maintenance are taxable to the payee spouse in the year received. Payments are deductible by the payor spouse, who does not have to itemize in order to receive the deduction. Child support is not taxable to the payee nor deductible by the payor. Again, the Tax Cuts and Jobs Act only applies to divorces that were finalized on or after 1/1/2019.

Tom was involved in a hurricane that was also a federally declared disaster that damaged his 1963 mint condition Chevy Corvette. Tom recently purchased the car for $36,000. Damages are estimated to be $20,000. Because Tom only intended to use the vehicle for special occasions, he did not fully insure it. The insurance company paid $5,000 towards the damages. Tom's adjusted gross income is $62,000. What amount can Tom deduct for the loss on his federal income tax return? A) $15,000. B)$0. C)$8,800. D)$8,700.

D // Casualty losses are limited to those incurred as a result of a federally declared disaster. Individual casualty losses on property not used for business are deductible only to the extent that the casualty loss exceeds $100 and 10% of the taxpayer's adjusted gross income. The casualty loss deduction is the lesser of the asset's basis or decrease in FMV. In this situation, the decrease in value is less than the basis of the asset. The calculation is: (Loss − insurance reimbursement − $100 in 2019 − 10% of AGI) or ($20,000 − $5,000 − $100 − $6,200) = $8,700

Max is the sole shareholder in the ABC Corporation. This year, ABC has accumulated earnings and profits of $40,000. Max's basis for the stock he owns in the company is $7,000. ABC makes a distribution of $60,000 to Max. How much of this distribution is taxable to Max as a capital gain? A)$7,000. B)$20,000. C)$0. D)$13,000.

D // Earnings and profits of the corporation are considered to be the first source of the distribution and are considered dividends. The first $40,000 is treated as a dividend. From the remaining $20,000, Max deducts his tax-free return of basis of $7,000. The net amount remaining of $13,000 will be treated as a capital gain.

Your client, Brenda, was subject to the alternative minimum tax (AMT) in prior years. She wants to ensure that she is not subject to the AMT this year. All of the following actions might increase Brenda's exposure to the individual AMT this year EXCEPT A)prepaying her real property taxes B)purchasing private activity bonds issued prior to January 1, 2009 C)gain on Section 1202 QSBS purchased after September 27, 2010 D)exercising nonqualified stock options

D // Exercising nonqualified stock options (not incentive stock options) would help Brenda avoid the individual AMT in the current year. Exercising such options would result in regular income tax and could, therefore, avoid the imposition of AMT because Brenda would pay the greater of regular income tax or individual AMT. Gain on Section 1202 QSBS purchased after September 27, 2010 is not included in regular income or AMT income.The other items are positive AMT adjustments or tax preference items.

Forty Brands, Inc., a C corporation, has averaged annual gross receipts of $35 million over the last 10 years. Mark owns 100% of the stock of Morgan Consulting, Inc., an actuarial science business for which he provides the reports to the clients. Becky owns a successful business, Hampton Street Auto Sales, selling vintage autos on consignment as a sole proprietor with average gross receipts of $10 million for all prior years. Jeff and Larry are partners in Tip Top Roofing that has gross receipts of $3 million. Which of the following entities may NOT use the cash method of accounting? A)Hampton Street Auto Sales. B)Tip Top Roofing. C)Morgan Consulting, Inc. D)Forty Brands, Inc.

D // Forty Brands, Inc. is a C corporation with annual gross receipts over $25 million and cannot use the cash method of accounting. A qualified personal service corporation (Morgan Consulting, Inc.) can use the cash method of accounting if services are provided in the fields of actuarial science, accounting, law, engineering, health, consulting, architecture, or performing arts and accurately reflect income. The other choices are incorrect because a sole proprietor may also use the cash method of accounting, and partnerships with gross receipts that do not exceed $25 million may use the cash method of accounting.

Jack would like to donate his collection of impressionist paintings to an impressionist art museum which is registered as a 501(c)3 public charity for educational purposes. The museum, in turn, will sell the paintings to generate income. Jack's original purchase price of the paintings was $50,000. The FMV of the paintings when he contributes them is $300,000. His adjusted gross income for the year is $1 million. Jack has come to you, a CFP® professional, for information regarding the tax effects of the proposed transaction. He has heard from friends that if the museum sells the collection, he will only be able to use basis to value the donation. After reviewing the documentation for the pending transfer, you tell Jack he will have a total deductible donation for the gift of: A)$250,000. B)$1 million. C)$0. D)$300,000.

D // Gifts of tangible personal property given to a charity are use-related if the gift will be used by the charity in a manner related to the purposes of that charity. The artwork given to the museum is use-related, even if the museum subsequently sells it. Gifts of use-related property are tax deductible for the FMV of the property when it is donated. Jack could deduct the full $300,000 value of the gift. As with other gifts, the donation is limited to 30% of the taxpayer's AGI for gifts of long term capital gain, use-related property to public charities. If Jack's AGI had been less than $1 million, the deduction would have been limited, but the excess amount of the gift could have been carried over to future years. If the museum were instead a private charitable foundation, the AGI limitation would be 20%.

Bob and Heather form a partnership. Bob contributes $30,000 in cash and $20,000 in stock certificates. Heather contributes $10,000 in cash and a tract of commercial real estate worth $40,000. Heather's basis in the real estate is $15,000. Bob and Heather are equal partners. Immediately after forming the partnership, Heather sells her interest in the partnership for $50,000. How much of a gain must Heather recognize for tax purposes on the sale of her partnership interest? A)$15,000. B)$75,000. C)$50,000. D)$25,000.

D // Heather's total basis in the partnership is equal to her basis in the property she contributed. Since she invested $10,000 in cash and real estate for which her basis was $15,000, her total basis in the partnership is $25,000. The sale of her partnership interest results in a gain of $25,000.

Kate was recently diagnosed with a terminal illness and was certified by her doctor on June 1 of the current year as terminally ill and expected to die within 24 months. On July 1 of the current year, Kate sold her life insurance policy with a face value of $500,000 to a viatical settlement provider for $340,000. Assuming she had paid $50,000 in premiums, how much of the $340,000 proceeds must Kate include in her gross income for the current year? A)$290,000. B)$340,000. C)$50,000. D)$0.

D // If the insured is terminally ill (expected to die within 24 months), proceeds received from a viatical settlement by the insured are not taxable.

Grace brought you a document she has executed stating that she is assigning the income from the ABC Mutual Fund shares that she owns to her son, Paul. Paul is facing some economic challenges and Grace believes the income from the mutual funds shares will help him. As Grace's planner, what do you tell her about this income assignment? A)The assignment of income will be valid if the mutual fund manager also signs the document of assignment. B)Grace can successfully assign the income to Paul because he is a family member. This option is unavailable to unrelated parties. C)Grace's document assigning the mutual fund income to Paul makes the income taxable to Paul and not Grace. D)Grace can assign the income to Paul but it is still taxable to Grace and not Paul because Grace still owns the mutual fund shares; she would be making a gift to Paul of the income.

D // In the analyzing and evaluating the client's current financial status, you have confirmed Grace's ownership of the mutual fund shares. Grace's attempt at assignment of income will not relieve her from the tax burden of the income from the shares. The assignment of income concept means that income is taxed to the individual who earns it. Therefore, an individual cannot avoid taxation simply by assigning income to another party or entity. In order for income not to be taxable, the source of the income must be gifted or sold to another party or entity prior to the income being earned. Once the property is gifted, the person gifting the property will have no income attributable from that source.

Last year, Marla sold 500 shares of ITN stock that she purchased 10 years ago. Her total purchase price of the stock was $500 and her total sales price was $1,000. She also sold 200 shares of ABC stock for $2,000 that she had purchased 8 years ago for $500. Finally, she sold 1,000 shares of BILCO stock for $10,000, for which she had paid $20,000 earlier in the year. What is Marla's net capital gain or loss in this situation? A)$2,000 loss. B)$2,000 gain. C)$8,000 gain. D)$8,000 loss.

D // Marla realized a long-term gain on the first two sales: $500 on the 1st and $1,500 on the 2nd. She realized a short-term loss of $10,000 on the 3rd. Her net capital loss for the year is $8,000 ($10,000 − $500 − $1,500).

Morgan, who is in the 35% marginal income tax bracket, recently made a loan of $200,000 to Taylor. The Federal interest rate was 4.3% at the time of the loan and Morgan charged Taylor 2%. During the first year of the loan, Taylor paid $333 in interest to Morgan. The interest at the Federal rate would have been $717. What amount of interest income Morgan must recognize in the first year of the loan? A)$384. B)$333. C)$0. D)$717.

D // Morgan has made a below-market interest rate loan to Taylor of $200,000. This is a gift loan for the difference between the interest charged to Taylor and the Federal interest rate. Because this loan was made for $200,000, it does not qualify for any of the exceptions to the imputed interest rules. Morgan must recognize the full $717 as interest income and Morgan has made a gift to Taylor of $384, the difference between the amount of interest paid and the Federal rate. This gift qualifies for the gift tax annual exclusion.

Julian, age 43, had the following income in the current year. He sold a building for $300,000 that had a basis of $100,000. The property had suspended passive losses of $50,000. Julian owns a portfolio of private activity bonds that paid interest of $25,000. Assuming he is single and has no other income, what is the largest deductible contribution he could make to a traditional IRA? A)$2,000. B)$6,000. C)$1,000. D)$0.

D // On the basis of the information given, Julian cannot contribute to an IRA because he does not have earned income.

Wickham, Inc. is a manufacturing concern organized as a C corporation. George is a potential investor in this closely-held business currently owned by 4 stockholders. George will be buying a 20% interest in the company. He has approached his planner to help him get a better idea of what kinds of taxes other than income tax will affect his investment. Which of the following statements regarding taxes and corporations is(are) CORRECT? 1. Because the corporation has 5 or fewer shareholders, it is subject to the personal holding company tax which is 20% in 2019. 2. Because of George's 20% ownership in the company, the dividends-received deduction applies to 65% of the dividends George receives from Wickham, Inc. 3. Wickham, Inc. could be subject to the accumulated earnings tax of 20% on undistributed earnings beyond its reasonable needs. 4. The stockholders of a closely-held corporation could be required to pay the excess net passive income tax on any dividends received from Wickham, Inc. A)4 only. B)1 and 3. C)2 and 4. D)3 only.

D // Only Statement 3 is correct. Statement 1 is incorrect because Wickham is a manufacturing concern with active, not passive, income and is therefore not subject to the personal holding company tax. Statement 2 is incorrect because the dividends-received deduction is available only to corporations, not to individuals. Statement 4 is incorrect because the excess net passive income tax applies only to S corporations.

On March 1 of this year, Barbara received a gift of commercial rental property from her mother, Jan. Jan's basis was $150,000 and the FMV was $200,000 on the date of the gift. Jan died on July 10 of this year and Barbara inherited Jan's IRA which was valued at $700,000 on the date of death. Barbara received $9,000 of rental income from the property for which she had $2,500 in expenses. Barbara received no distributions from the IRA this year but will begin taking distributions next year using her own life expectancy. Which of the following statements regarding the properties Barbara received from Jan is(are) CORRECT? 1. Because Jan died within 3 years of the gift to Barbara, the commercial rental property becomes part of Jan's gross estate. Barbara still reports the rental income and expenses on her own tax return. 2. Barbara must pay both income tax and estate tax on the IRA she inherited from Jan. 3. The IRA is income in respect of a decedent (IRD) and there is a miscellaneous itemized deduction allowed for the portion of estate tax attributed to the IRD. 4. Because Barbara cannot show a profit on renting the commercial rental property in 3 of the last 5 years, it will be treated as a hobby for income tax purposes. A)2 and 3. B)4 only. C)1, 2, 3, and 4. D)3 only.

D // Only Statement 3 is correct. The IRA is considered IRD and estate tax paid on the IRA is allowed as a deduction on the beneficiary's income tax return for the proportional share attributed to the value of the IRA. Statement 1 is incorrect. Only the gift tax paid on gifts made within 3 years of Jan's death is included in the gross estate. Barbara does still report the rental income and expenses on her own income tax return. Statement 2 is incorrect. Barbara will only pay income tax on the distribution she receives from the IRA next year. The estate tax is paid by the estate. Statement 4 is incorrect. Rental property is not treated as a hobby, but will more likely be treated as a passive activity unless Barbara is a real estate professional.

Philip, a professor, earned a salary of $140,000 from a university in the current year. He received $35,000 in dividends and interest during the year. In addition, he incurred a loss of $25,000 from an investment in a passive activity. His at-risk amount in the activity at the beginning of the current year was $15,000. What is Philip's adjusted gross income (AGI) for the current year? A)$115,000. B)$160,000. C)$150,000. D)$175,000.

D // Philip's AGI, after considering the passive investment, is $175,000 ($140,000 active income + $35,000 portfolio income). He cannot offset the passive loss against active or portfolio income. The loss may be deducted only against passive income, which he does not have in the current year.

Joe and Cathy are married and are active participants in rental real estate. They have earned income of $125,000, $5,000 of unearned income, $30,000 in deductions, and $20,000 in losses related to their real estate holdings. How much will the couple claim as taxable income? A)$85,000. B)$80,000. C)$95,000. D)$90,000.

D // Rental and real estate passive losses are allowed up to $25,000 for taxpayers not filing as MFS, which can be used to offset non-passive income. The $25,000 is reduced (but not below zero) by 50% of the amount by which the taxpayer's AGI exceeds $100,000. Joe and Cathy have income of $130,000. The passive real estate maximum allowable loss deduction must be reduced by $15,000 to $10,000 or [(130,000 − 100,000) × .5]; therefore, the maximum rental activity passive loss they can claim is $10,000 ($25,000 − $15,000). The couple's taxable income is $130,000 less deductions and allowable losses of $40,000 ($30,000 + $10,000), which leaves a taxable income of $90,000. Note that any unused loss may be carried forward. Note here that if the loss was $9,000, it would have been entirely deductible because it is below the adjusted allowable maximum. The allowable maximum deduction is calculated before considering how much of the loss is deductible.

Lydia plans to gift assets to family and friends as part of her tax and estate planning. She is concerned about the income tax effect the transfers will have on the recipients. Some of the assets she is considering are income-producing assets and others are simply investments which will yield cash only upon disposal by the recipient. She has asked her financial planner for recommendations on transferring the assets with a minimum impact on the recipients' income tax situations while also providing tax advantages for herself. Which of the following is(are) appropriate recommendation(s) her planner can suggest to her? 1. Lydia should gift loss property immediately to her best friend, Carol. 2. An office building for which Lydia's basis is $100,000 and the FMV is $175,000 should be transferred via her will to her designated heir. 3. Appreciated stock should be gifted immediately so the recipient will receive a stepped-up basis. A)2 and 3. B)1 only. C)1 and 2. D)2 only.

D // Statement 1 is incorrect because Lydia should sell the loss property so she can take advantage of the loss on her tax return. She may gift the cash realized. Statement 2 is correct because inherited property generally receives a step-up in basis to FMV at the date of death, increasing the basis to the heir. If it were gifted instead, the donee would have the same basis as Lydia and an increased capital gain upon disposal. Statement 3 is incorrect because the donee receives a carryover basis in gifted property, not a step-up.

Arthur, an attorney, owns and participates in a separate business (not real estate) during the current year. He has one employee who works part time in the business. Which of the following statements is CORRECT? 1. If Arthur participates for 500 hours and the employee participates for 520 hours during the year, Arthur qualifies as a material participant. 2. If Arthur participates for 600 hours and the employee participates for 1,000 hours during the year, Arthur qualifies as a material participant. 3. If Arthur participates for 120 hours and the employee participates for 125 hours during the year, Arthur qualifies as a material participant. 4. If Arthur participates for 95 hours and the employee participates for 5 hours during the year, Arthur probably does not qualify as a material participant. A)1 only. B)3 only. C)4 only. D)2 only.

D // Statement 2 is correct. An individual who participates for more than 500 hours is a material participant regardless of how much the employees of the business participate. Statement 1 is incorrect. Arthur would have to participate for more than 500 hours for this to be correct. Statement 3 is incorrect. Arthur participates for more than 100 hours, and this is not more than the participation of any other individual. Statement 4 is incorrect. Arthur's participation constitutes substantially all of the participation, even though Arthur's participation is less than 100 hours.

Jenny graduated from college with an engineering degree this year. Last year, her only income was approximately $25,000 from a part-time job. This year, she will begin working as an engineer, and she expects her salary for the year to be $150,000. In addition, last week she won a lump-sum prize of $500,000 in the lottery. She lives in a state with a flat-rate income tax. Which of the following statements regarding her income tax status for the current year is(are) CORRECT? i. Her marginal federal income tax rate will probably be higher than last year. ii. Her marginal state income tax rate will probably be higher than last year. A)Both I and II B)II only C)Neither I or II D)I only

D // Statement I is correct. The federal income tax system is a progressive tax system, so the significant increase to income will probably push her into a higher marginal tax bracket. Statement II is incorrect. Because Jenny lives in a state with a flat-rate income tax, her increased income will not push her into a higher marginal tax bracket.

Valerie and her husband, Mark, own a used car business. This year, they purchased a parcel of raw land for business expansion. They paid $150,000 for the land, incurred legal fees of $4,500 associated with the purchase, and paid a broker $4,000 for his services. What is their adjusted basis in the land? A)$150,000. B)$154,000. C)$154,500. D)$158,500.

D // The adjusted basis is calculated by starting with the original purchase price ($150,000) and increasing it by certain allowable costs. In this case, the allowable costs to increase basis are legal fees and broker commission (4,000 + 4,500). Therefore, the adjusted basis is 150,000 + 8,500 = 158,500.

Jack purchases a $100,000 machine for his clothing factory. The machine has a useful life of 20 years. The expected salvage value of the machine is $3,000 at the end of its useful life. How much of a tax deduction is Jack entitled to take during the first year the asset is in use, if he is using the straight-line method of depreciation? A)$4,850. B)$5,000. C)$3,750. D)$2,425.

D // The annual depreciation deduction under the straight-line method is the adjusted basis of the property at the beginning of the taxable year, less the salvage value divided by the remaining useful life of the asset. In this case, the adjusted basis for the first year is $100,000 and the salvage value is $3,000, for a total of $97,000 to be apportioned over 20 years, or $4,850 per year; however you must remember to use the half-year convention. This results in a deduction of $2,425.

Jeff is retiring and will begin receiving monthly annuity payments from his employer on November 1st of this year. Jeff is 66 years old and has contributed $151,200 to the annuity. The annuity payments will be $1,200 per month and will continue for his life. Based on his age, Jeff's designated number of annuity payments are 210 months. How much of this year's annuity payments will be taxed as ordinary income? A)$2,400. B)$1,440. C)$5,760. D)$960.

D // The excludable portion is calculated by multiplying each payment received by an exclusion ratio. In this situation, the exclusion ratio is calculated by dividing the $151,200 investment by 210 annuity payments. Accordingly, the exclusion ratio is 60% ($151,200 ÷ $252,000). Therefore, 60% of each annuity payment ($720) is considered a return of capital and $480 is taxable. Because only 2 annuity payments will be received this year, $960 will be included as ordinary income in Jeff's gross income.

Betty and Wilma are friends and business associates. Betty gives Wilma a piano that has a FMV of $10,000. In exchange, Wilma excuses a $15,000 debt that Betty owes her. If Betty's original purchase price of the piano was $4,000, what is her realized gain in this transaction? A)$10,000. B)$15,000. C)$6,000. D)$11,000.

D // The gain realized is the difference between the amount realized and the adjusted basis. Betty had a total cost basis in the piano of $4,000. The FMV of the piano when she gave it to Wilma was $10,000. If Betty had instead sold the piano at that time, she would have had a net gain of $6,000. In addition to this gain, she had a $15,000 debt excused for the price of the $10,000 piano. She not only earned the $6,000, but also received an additional $5,000 in non-cash consideration for a total gain of $11,000.

In the current year, Heather purchased $500,000 of common stock in ADM Corporation. To purchase the stock, she borrowed the $500,000 from her bank. During this year, she received $25,000 in dividend payments from the stock and paid $35,000 in interest on the loan. How much of an investment interest deduction will Heather be allowed to take on her tax return for this year? A)$10,000. B)$0. C)$35,000. D)$25,000.

D // The maximum deduction allowed for interest related to an investment debt is the taxpayer's net investment income. Because Heather received dividend income of $25,000, that is her net investment income; therefore, she can only deduct $25,000 of the interest payment this year. She can carry the excess deduction of $10,000 over to a future year.

Mark is forming a new business and is seeking investors. He is considering using either an S corporation or a limited liability company. He would like to limit the investors' liability to creditors. His father, Eric, who has a grantor retained annuity trust, has agreed to buy shares of the S corporation stock and contribute the shares to his GRAT as a good income generator for the trust. Mark's cousin, Karl, is citizen and resident of Germany and is also interested in investing in shares of the business. Mark's sister, Sara, would like to buy shares through her partnership, Road Crew, to support her brother's business venture. Candace is Mark's friend who is also interested in buying shares in the business. Mark has discussed this list of potential investors with his planner and his planner sees some potential problems. What should Mark's planner tell him? A)Sara and Karl may be investors if the entity form is an S corporation but not in a partnership. B)Because GRATs can only hold regular corporation shares, Eric cannot use it to invest in the S corporation. C)Only Mark, Candace and Eric (through his GRAT) may be investors in the business, no matter which entity form Mark chooses. D)Mark should choose the limited liability company entity form form to accommodate all of his investors.

D // The only entity form that will limit investor liability and accommodate all of Mark's investors is the limited liability company form. Karl is a nonresident alien and cannot be a shareholder in an S corporation. Sara's partnership is also prohibited from being an S corporation investor. If the owner of a GRAT may invest in a business entity, then the GRAT may also invest as the grantor is treated as the owner of the GRAT for income tax purposes. A limited liability company taxed as a partnership would limit liability for all of the investors in the business. While a limited partnership form would protect the any limited partners, there must be at least one general partner who would have unlimited liability.

Ward is a stock broker who is also a CFP© professional. Ted comes to him to discuss possible investments and ways to reduce his taxes on the income from those investments. Ted is interested in tax-exempt municipal bonds but Ward is discouraging. His commission will be much less on the bonds Ted is interested in than on other stock offerings Ward would like Ted to choose. Ward persuades Ted to buy stock and put the shares with a potential dividend payout this year of $10,000 in his 4-year-old son's name. Ward explains that the stock dividends would be taxed at the son's 0% rate and not Ted's 15% income tax rate for qualified dividends. What is the effect of Ward's advice to Ted? A)The gift of the stock shares would be subject to the kiddie tax rules. B)Both Ward and Ted have achieved their goals in this transaction. C)Ward has violated the Principle of Integrity but not the Principle of Fairness. D)A portion of the stock dividends would be taxed at trust and estate tax rates.

D // The stock dividends from the shares gifted to Ted's son are subject to the kiddie tax rules. Dividends in excess of the $2,200 (for 2019) kiddie tax threshold will be taxed to the child at the trust and estate tax rates. The gift of the shares is subject to the gift tax rules and not the kiddie tax rules. Ward has violated the Principles of Integrity, Fairness, Objectivity, and Competence by persuading Ted to buy stock that provides Ward with a higher commission to Ward's own advantage, not disclosing a conflict of interest, not providing services objectively, and without the knowledge of the correct kiddie tax rules. Only Ward has achieved his goal in the transaction. Ted's goal of reducing the taxes is only marginally achieved by the $1,100 of dividends taxed at the son's 0% tax rate in 2019. Interest from the tax-exempt municipal bonds would have been excluded from income for regular income tax purposes.

In January 10 of last year (Year 1), Todd sold stock with a cost of $6,000 to his son, Trey, for $4,000 (its fair market value). On July 31 of the current year (Year 2), Trey sold the same stock for $5,000 in a bona fide arms-length transaction to Mary, who is unrelated to Trey or Todd. What is the proper tax treatment of these transactions? A)Todd has a recognized loss of $2,000 in Year 1, and Trey has a recognized gain of $1,000 Year 2. B)Trey has a recognized gain of $2,000 in Year 2. C)Trey has a recognized gain of $1,000 in Year 2. D)Neither Todd nor Trey has a recognized gain or loss in either Year 1 or Year 2.

D // Todd's sale to Trey, his son, is a related-party transaction, and therefore, Todd cannot recognize a loss ($4,000 − $6,000). Trey's basis in the stock is $4,000. When he sells it for $5,000, he has a realized gain of $1,000. However, Trey will not recognize this gain because he can utilize $1,000 of his father's unrecognized loss. The remaining $1,000 of Todd's loss is gone forever.

Harvey is a one-third partner in the HARVMAN partnership. Harvey's adjusted basis in the partnership is $10,000. Harvey decides to sell his partnership interest for $40,000. Assuming the partnership has no unrealized receivables and no substantially appreciated inventory, how will the sale of the partnership be treated on Harvey's personal income tax return in the year he makes the sale? A)He must recognize ordinary income of $30,000. B)He must recognize a net capital gain of $40,000. C)He must recognize ordinary income of $40,000. D)He must recognize a net capital gain of $30,000.

D // When a partner sells his interest in a partnership, the gain is treated as a capital gain, unless the partnership has substantially appreciated inventory or unrealized receivables. The net capital gain equals the sale price less the partner's basis, which in this case is $40,000 less $10,000, or $30,000.

Jenny, a dentist in Georgia, has owned her office building for 15 years. It currently has a FMV of $400,000. She has an adjusted basis in the building of $100,000. She decides to move her dental practice to Iowa and finds a dentist there who wishes to move to Georgia. They agree to trade ownership of their respective office buildings. The value of the office building in Iowa is $600,000 and Jenny gave $200,000 boot in the exchange in cash. If Jenny later decides to retire from dentistry and she sells the office building for $1,000,000, how much of a gain must Jenny recognize (ignore any depreciation)? A)$100,000. B)$0. C)$400,000. D)$700,000.

D // When properties are traded and the use for the newly acquired property is the same as for the old property, the Tax Code does not require the taxpayer to recognize any gain received in the exchange. The gain can be carried over into the future. When the taxpayer does sell the property, she must use the basis that would have been used in the exchange. Because Jenny recognized no capital gain when she exchanged the property but gave $200,000 of cash, her basis is still $300,000 (assuming no adjustments to basis from depreciation) and she must then recognize a capital gain of $700,000.

On October 15, 2018, Erin purchased stock in Glennan Irish Ale Corporation for $2,000 (the stock is not small business stock). On June 15, 2019, the stock became worthless. How should Erin treat the loss in 2019? A)$1,000 long-term capital loss. B)$2,000 short-term capital loss. C)$1,000 short-term capital loss. D)$2,000 long-term capital loss.

D // Worthless securities are treated as becoming worthless at year end. Therefore, the loss is a long-term capital loss even though the stock became worthless after only 8 months.

Margo is considering selling property she inherited from her mother's estate. She has been living in the home she inherited for the last 3 years. She also has some inherited personal property that she wants to sell, some of which has increased in value while other items have lost value. She has questions about the income she will realize on the sale of these assets and has come to her CFP® professional, Paul, for answers. Which of the following statements that Paul makes to Margo regarding realized gain is NOT correct? A)As a general rule, there must be a taxable event that brings the income into the taxpayer's possession. B)A gain is generally not recognized from property transactions until there is a disposition of the property. C)An individual taxpayer who uses the cash basis method of tax accounting is not taxed on income until the income is actually or constructively received. D)Realized gain is always taxable.

D //Realized gain on the disposition of property is not always taxable. There are exclusions and deferrals in the Tax Code that may prevent recognition of all or part of the realized gain as includable in gross income (such as a Section 121 exclusion of a certain amount of the gain on the sale of a personal residence.)


Related study sets